SCI - Neurology Flashcards

1
Q
  1. A 52-year-old man presents with a 6 month history of progressive weakness. He first noticed difficulty with fine motor coordination of the right hand, but this later progressed to involve the right arm and later all extremities. On examination, he has dysarthria and reduced gag reflex. There is generalized motor weakness and atrophy. Deep tendon reflexes are hyperactive in both upper and lower extremities. Sustained ankle clonus is present. Fasciculations are seen in the tongue, across the chest, and in all limbs proximally. Sensory examination is normal. There is no family history of neurologic disorders. 1. What is the most likely diagnosis in this patient? a. Progressive muscular atrophy b. Amyotrophic lateral sclerosis c. Primary lateral sclerosis d. Adult-onset spinal muscular atrophy e. Kennedy’s disease (X-linked spinobulbar muscular atrophy) 2. Which of the following statements is correct regarding the disorder depicted in question 1? a. Dysphagia and dysarthria are rare as presenting features and do not typically occur as the disease progresses b. Acute respiratory failure is a common initial presentation c. Urinary incontinence and impotence occur early d. Cognitive dysfunction is present in many patients with this disorder e. Pseudobulbar affect (pathologic laughter and crying) is not a feature of this disorder, and its presence should prompt evaluation for other causes
A
  1. b, 2. d This patient’s history and examination, including evidence of upper and lower motor neuron dysfunction, are suggestive of amyotrophic lateral sclerosis (ALS). ALS is a neurodegenerative disorder that affects motor neurons in the anterior horn of the spinal cord, but also the motor cortex and brain stem. Sporadic ALS affects men more than women. It can present at any age, but with a peak incidence in the sixth to seventh decades of life. The hallmark of ALS is evidence of both upper motor neuron (UMN) disease (such as hyperreflexia, clonus, and presence of a Babinski sign) and lower motor neuron (LMN) disease (atrophy, fasciculations in the limbs, trunk, tongue, and occasionally face) in the absence of sensory symptoms or sensory abnormalities on examination. Clinical presentation is variable, but most often muscle weakness begins focally, usually in the extremities (two-thirds of cases) and spreads to involve contiguous regions, though there is significant variability in the pattern of motor weakness. The split-hand phenomenon is a feature of ALS, characterized by weakness and atrophy of the lateral hand (thenar and first dorsal interosseous muscles) with relative sparing of the medial hand (hypothenar) muscles. About one-third of patients present with bulbar symptoms, such as dysarthria or dysphagia (bulbar-onset ALS). Other symptoms of bulbar involvement include sialorrhea. Pseudobulbar palsy (pathologic laughter and crying without a corresponding change in mood) and excessive yawning also occur in ALS and do not necessarily suggest an alternative diagnosis. Muscle cramps commonly occur. Only a minority of patients with ALS (approximately 1%) present with acute respiratory failure, though the majority progress to respiratory failure. Cognitive impairment of some degree is present in up to 50% of patients with ALS; it is mostly subclinical, but can be detected on neuropsychologic testing. In a small proportion of cases, the cognitive dysfunction is more evident, manifesting as a dementia that is typically of the frontotemporal type, namely frontotemporal dementia (FTD). This tends to occur most frequently in patients who present with bulbar-onset ALS. On the other hand, a subset of patients presenting with FTD have clinical and electrodiagnostic features of motor neuron disease. The pathology of patients with so-called ALS-FTD includes atrophy in the frontal and temporal lobes and ubiquitin-positive, TDP-43-positive, tau-negative inclusions. Autosomal dominant forms have been linked to TDP-43 gene mutations on chromosome 9. Curiously (for unclear reasons), the region of motor neurons in the sacral spinal cord that are responsible for sphincter control, Onufrowicz nucleus (also known as Onuf’s nucleus) is generally spared in ALS, and sphincter dysfunction is typically not a prominent problem. Sensory involvement and dysautonomia are also uncommon in ALS, but can occur. ALS is relentlessly progressive and ultimately fatal within 2 to 5 years of symptom onset, although in approximately 20%, survival is longer than 5 to 10 years after symptom onset. Primary lateral sclerosis (discussed in question 6), progressive muscular atrophy (discussed in question 10), Kennedy’s disease (X-linked spinobulbar muscular atrophy, discussed in question 33), and spinal muscular atrophy (discussed in question 35) are all motor neuron disorders. In these disorders, either UMNs or LMNs alone are affected, not in combination, in contradistinction to ALS. In early ALS, however, combined UMN and LMN dysfunction may be subtle or absent until later stages when its presence is necessary for diagnosis (by definition; see question 31). In Kennedy’s disease, there is often a family history of motor neuron disease in males and gynecomastia is present on examination. Bradley WG, Daroff RB, Fenichel GM, et al. Neurology in Clinical Practice, 5th ed. Philadelphia, PA: Elsevier; 2008.
How well did you know this?
1
Not at all
2
3
4
5
Perfectly
2
Q
  1. A 16-year-old boy presents with progressive bilateral lower extremity numbness and weakness, with difficulty walking for the past 2 months. He reports worsening over the past 5 days, and this morning when he woke up he was unable to move his legs and felt numb from his upper chest down. His examination shows mild weakness and areflexia in both upper extremities, and significant weakness with spasticity and hyperreflexia in the lower extremities. He has a sensory level at T2. His MRI is show in Figure 11.1. Which of the following is the most likely diagnosis?

a. Acute transverse myelitis
b. Spinal cord infarct
c. Cord compression from a metastatic tumor
d. Epidural hematoma
e. Meningioma with spinal cord compression

A
  1. e
    Figure 11.1 shows an intradural extramedullary tumor compressing the spinal cord. The tumor is located at the C7 to T1 vertebral level and is dural based. Meningiomas are one of the most common intradural extramedullary tumors. They are nonglial neoplasms that are usually benign and can cause neurologic problems due to compression. Because they grow slowly, neurologic manifestations progress very gradually and may compensate for the degree of compression. Patients may have very large tumors causing significant compression without prominent manifestations on clinical examination.
    This patient does not have an acute transverse myelitis nor a spinal cord infarct because the presentation in both of these conditions is usually acute to subacute, with MRI findings showing a parenchymal cord lesion and not an extramedullary mass. Spinal cord infarct may be sudden in onset and usually affects the anterior cord (anterior spinal artery), causing primarily bilateral weakness and pain and temperature loss rather than vibration and position loss (dorsal columns are relatively spared). Acute transverse myelitis progresses over hours or a few days rather than months. A metastatic tumor usually has a more subacute presentation with prominent back pain. Because metastatic tumors develop more rapidly, the patient would have more symptoms and neurologic deficits than this patient has for the degree of compression seen on this MRI. An epidural hematoma also presents rapidly with an acute compressive myelopathy syndrome manifesting as spinal shock, usually with no evidence of spasticity or hyperreflexia on presentation.
    Bradley WG, Daroff RB, Fenichel GM, et al. Neurology in Clinical Practice, 5th ed. Philadelphia, PA: Elsevier; 2008.
    Jacob A, Weinshenker BG. An approach to the diagnosis of acute transverse myelitis. Semin Neurol. 2008; 28:105–120.
    Ropper AH, Samuels MA. Adams and Victor’s Principles of Neurology, 9th ed. New York: McGraw-Hill; 2009.
How well did you know this?
1
Not at all
2
3
4
5
Perfectly
3
Q
  1. Which of the following statements is incorrect regarding the management of amyotrophic lateral sclerosis (ALS)?
    a. Periodic assessment of lung function is indicated in patients with ALS
    b. There is evidence to suggest that management of patients with ALS in multidisciplinary clinics is beneficial
    c. Riluzole is an inhibitor of glutamate release that modestly slows disease progression
    d. Frequent evaluation of swallowing function is important as patients with ALS are at increased risk of aspiration
    e. Parotid and submandibular botulinum toxin injections are ineffective in patients with ALS with refractory sialorrhea
A
  1. e
    Parotid and submandibular botulinum toxin injections are effective in some patients with amyotrophic lateral sclerosis (ALS) with refractory sialorrhea, and should be offered when appropriate.
    Guidelines have been issued by the American Academy of Neurology for the management of patients with ALS. The majority of patients experience hypoventilation, and orthopnea becomes significant in advanced stages. Nocturnal oximetry, supine forced vital capacity, and supine maximal inspiratory pressure should be periodically assessed in patients with ALS. Noninvasive ventilation should be offered as indicated, and it likely improves quality of life. Management of patients with ALS in multidisciplinary clinics, including experts in neurology, speech therapy, physical and occupational therapy, respiratory therapy, social work, and case management, is likely of benefit.
    Pharmacologic treatment of ALS may include riluzole, an inhibitor of glutamate release, which slows disease progression, prolonging survival by approximately 3 months. Patients may experience side effects of dizziness, fatigue, and gastrointestinal upset, which are usually mild and transient; if intolerable, dosage can be reduced or taken with food prior to deciding to discontinue it. Importantly, blood count and liver enzymes should be checked routinely 1 month after beginning riluzole, monthly for 3 months, then every 3 months for the rest of the first year, and yearly afterward. Other medications being investigated as disease-modifying therapies include ceftriaxone, creatine monohydrate, and tamoxifen, but definitive data showing benefit are not available as of 2010. Management is therefore largely symptomatic.
    Frequent evaluation of swallowing function is important as patients with ALS are at increased risk of aspiration. Placement of a percutaneous gastrostomy tube stabilizes body weight and maintains hydration in patients with ALS; although survival is probably not prolonged significantly, quality of life is improved. Treatment of pseudobulbar affect is off-label with tricyclic antidepressants or selective serotonin reuptake inhibitors, although an effective dextromethorphan-quinidine combination was FDA approved in October 2010.
    Miller RG, Jackson CE, Kasarskis EJ, et al. Quality Standards Subcommittee of the American Academy of Neurology. Practice parameter update: The care of the patient with amyotrophic lateral sclerosis: drug, nutritional, and respiratory therapies (an evidence-based review): Report of the Quality Standards Subcommittee of the American Academy of Neurology. Neurology. 2009; 73:1218–1226.
    Milller RG, Jackson CE, Kasarskis EJ, et al. Quality Standards Subcommittee of the American Academy of Neurology. Practice parameter update: The care of the patient with amyotrophic lateral sclerosis: Multidisciplinary care, symptom management, and cognitive/behavioral impairment (an evidence-based review): Report of the Quality Standards Subcommittee of the American Academy of Neurology. Neurology. 2009; 73:1227–1233.
How well did you know this?
1
Not at all
2
3
4
5
Perfectly
4
Q
  1. A patient with a chronic progressive myelopathy presents for evaluation. You suspect vitamin B12 deficiency. Regarding this condition, which of the following is incorrect?
    a. Vitamin B12 levels may be within normal limits despite clinical manifestations of deficiency
    b. Elevated gastrin levels, anti-intrinsic factor antibodies, and anti-parietal cell antibodies may be associated with vitamin B12 deficiency
    c. It can cause white matter vacuolization
    d. Serum vitamin B12 level is the only laboratory test that needs to be followed when treating these patients
    e. The most common cause of vitamin B12 deficiency is malabsorption
A
  1. d
    Serum methylmalonic acid (MMA) and homocysteine levels should be obtained when following patients with vitamin B12 deficiency receiving treatment.
    Vitamin B12 or cobalamin deficiency is a major cause of neurologic disease. Manifestations include subacute combined degeneration of the spinal cord, peripheral neuropathy, optic neuropathy, and even cognitive impairment.
    Vitamin B12 as methylcobalamin is a cofactor for the enzyme methionine synthase in the conversion of homocysteine into methionine, which is then adenylated to S-adenosylmethionine. S-adenosylmethionine is required for methylation reactions, and decreased levels may lead to reduced myelin basic protein methylation and white matter vacuolization. Methionine also facilitates the formation of formyltetrahydrofolate, which is involved in purine and pyrimidine synthesis, and therefore low methionine levels secondary to vitamin B12 deficiency impair DNA synthesis. In another pathway, vitamin B12 as adenosylcobalamin is a cofactor in the conversion of methylmalonyl CoA to succinyl-CoA. Vitamin B12 deficiency will lead to the accumulation of methylmalonate and propionate, providing abnormal substrates for fatty acid synthesis, and therefore interfering with myelin synthesis.
    In some cases of vitamin B12 deficiency, vitamin B12 levels may be within normal limits; however, MMA and homocysteine will be elevated, helping in the diagnosis.
    Malabsorption is the most common cause of vitamin B12 deficiency, and achlorhydria in the setting of pernicious anemia is an important cause. Pernicious anemia is an autoimmune condition in which anti-intrinsic factor and anti-parietal antibodies may be present. Given that these patients have achlorhydria, gastrin levels may be elevated as well. Parenteral vitamin B12 supplementation is the treatment of choice in cases of malabsorption, and usually the hematologic abnormalities improve rapidly; however, the neurologic manifestations may not resolve completely. Vitamin B12 levels rise with treatment, regardless of the clinical effect; therefore, MMA and homocysteine levels should be followed.
    Ropper AH, Samuels MA. Adams and Victor’s Principles of Neurology, 9th ed. New York: McGraw-Hill; 2009.
How well did you know this?
1
Not at all
2
3
4
5
Perfectly
5
Q
  1. A 52-year-old woman from Ohio who has not traveled outside of the state her entire life presents with gait difficulties of 2 years’ duration. There is no family history of a similar problem. On examination, she has significant lower extremity spasticity, with a scissoring gait. There is bilateral ankle clonus, Babinski signs are present bilaterally, and deep tendon reflexes are pathologically brisk throughout, especially at the knees and ankles. There are no fasciculations. Sensory examination is normal. MRI of the brain and spine are normal. What is the most likely diagnosis in this patient?
    a. Tropical spastic paraparesis

b. Amyotrophic lateral sclerosis
c. Hereditary spastic paraparesis
d. Primary lateral sclerosis
e. Cervical spondylosis

A
  1. d
    Primary lateral sclerosis (PLS) is characterized by presence of upper motor neuron signs at least 3 years from symptom onset, without evidence of lower motor neuron dysfunction. PLS is considered along the spectrum of amyotrophic lateral sclerosis (ALS), although it is controversial as to whether or not it is a separate entity. It typically presents in the sixth decade of life with a progressive spastic tetraparesis and later cranial nerve involvement. Rarely, bulbar onset occurs. Spasticity, rather than muscle weakness or atrophy, is the most prominent feature. It typically progresses slowly over years. Autonomic involvement does not typically occur. Autopsy on patients with PLS has shown significant cell loss in layer 5 of the motor and premotor cortex, predominantly the large pyramidal Betz cells with corticospinal tract degeneration. The majority of patients with ALS who present with predominantly upper motor neuron signs and symptoms eventually develop lower motor neuron involvement within 3 to 4 years.
    The differential diagnosis of PLS is broad and a thorough work-up to exclude any possibilities is indicated. It includes cervical myelopathy, although sensory abnormalities and other features of myelopathy such as bowel and bladder symptoms should be present, and a normal MRI of the spine excludes this. Similarly, multiple sclerosis is high on the differential diagnosis of PLS; the presence of sensory symptoms, changes on MRI of the brain and spine, and CSF findings (when CSF is available for analysis) distinguish between the two. Tropical spastic paraparesis due to HTLV-1 typically occurs in endemic regions (equatorial and south Africa, the Caribbean, parts of Asia, and Central and South America) and rarely in the United States. Patients with hereditary spastic paraparesis often, though not always, have a family history of similar symptoms and signs and have white matter abnormalities on MRI of the brain and spine. Another differential diagnosis is stiff person syndrome, which is associated with anti-glutamic acid decarboxylase antibodies, which should be assessed for in a patient with this presentation. Other disorders that can present with spastic quadriparesis that should be considered include hexosaminidase A deficiency, adrenomyeloneuropathy, a paraneoplastic process, and autoimmune processes (such as Sjgren’s syndrome).
    Treatment is symptomatic, and usually includes baclofen (a GABAB agonist) or tizanidine (an α2 agonist) to reduce spasticity.
    Bradley WG, Daroff RB, Fenichel GM, et al. Neurology in Clinical Practice, 5th ed. Philadelphia, PA: Elsevier; 200
How well did you know this?
1
Not at all
2
3
4
5
Perfectly
6
Q
  1. A 52-year-old man who is an intravenous drug user, HIV seropositive with a CD4 count of 120 and noncompliant with antiretrovirals, develops progressive neurologic symptoms over several months, including ataxia, spastic paraparesis, and sensory loss below his waist. After you examine him you suspect that he has a myelopathy. Which of the following is correct regarding HIV-related myelopathy?
    a. Copper deficiency is the most important factor for the development of this type of myelopathy
    b. Vitamin B12 deficiency is the most important cause of this type of myelopathy
    c. Pathologic analysis of the spinal cord shows lateral and posterior column demyelination with microvacuolar changes
    d. Pathologic analysis of the spinal cord shows axonal degeneration as the hallmark finding
    e. HTLV-1 is the most common etiology associated with this type of myelopathy
A
  1. c
    This patient has HIV, is immunocompromised as evidenced by the low CD4 count, and has a progressive myelopathy. This population of patients should undergo an evaluation for various infectious agents as the potential cause of myelopathy, including syphilis, viruses such as herpes zoster and HTLV-1, mycobacteria, and fungal agents. However, the most common cause of spinal cord involvement in patients with HIV is primary HIV-related myelopathy. This is a vacuolar myelopathy, in which there is lateral and posterior column demyelination with microvacuolar changes and axonal preservation. Given that other potential conditions can cause myelopathic symptoms in patients with HIV, HIV-related vacuolar myelopathy should be a diagnosis of exclusion.
    HIV-related vacuolar myelopathy presents in patients with AIDS, and affected patients have gradually progressive gait difficulty, spasticity, leg weakness, and impaired proprioception. There is no back pain and the upper extremities are typically spared. MRI may reveal spinal cord atrophy, but is most often normal. There is no effective treatment, and the initial goal is prevention with anti-retroviral therapy.
    HTLV-1 is a cause of spinal cord disease in patients with HIV; however, it is not the cause of HIV-related vacuolar myelopathy. HTLV-1 causes tropical spastic paraparesis or HTLV-1-associated myelopathy. This is a chronic progressive myelopathy endemic in equatorial and south Africa, as well as in parts of Asia, Central and South America, and the Caribbean. This virus can be transmitted via contaminated blood, sexual activity, breast-feeding, and rarely in utero. Only 1% to 2% of infected individuals will develop neurologic disease. These patients will manifest with a slowly progressive spastic paraparesis, lower extremity paresthesias, painful sensory neuropathy, and bladder dysfunction. MRI reveals increased signal on T2-weighted images with atrophy of the thoracic cord, but these findings are not specific for this condition. The diagnosis is made with positive serology in blood and CSF to HTLV-1, as well as PCR. No antiviral agent is available for the treatment of HTLV-1.
    Boisse L, Gill J, Power C. HIV infection of the central nervous system: Clinical features and neuropathogenesis. Neurol Clin. 2008; 26:799–819.
    Bradley WG, Daroff RB, Fenichel GM, et al. Neurology in Clinical Practice, 5th ed. Philadelphia, PA: Elsevier; 2008.
    Ropper AH, Samuels MA. Adams and Victor’s Principles of Neurology, 9th ed. New York: McGraw-Hill; 2009.
How well did you know this?
1
Not at all
2
3
4
5
Perfectly
7
Q
  1. A 30-year-old man presents for evaluation of gradually progressive lower extremity spasticity, which has worsened slowly over the past 5 years. On examination he has spasticity of the lower extremities, with no evidence of lower motor neuron findings and no other neurologic abnormalities. He says that his father had a similar problem, and his younger brother is starting to have difficulty walking. Work-up has been negative, including an MRI of the brain and cervicothoracic spine that was unremarkable, normal vitamin B12 and copper levels, normal CSF studies for infectious and inflammatory conditions, negative HIV and HTLV-1 serologies, and nonreactive VDRL. Which of the following is the most likely diagnosis?
    a. Hirayama disease
    b. Hereditary spastic paraparesis
    c. Amyotrophic lateral sclerosis
    d. Progressive muscular atrophy
    e. Multiple sclerosis
A
  1. b
    On the basis of the family history and clinical findings of this patient, he most likely has hereditary spastic paraparesis (HSP), which is a group of disorders characterized by progressively worsening spasticity of the lower extremities, with variable weakness and difficulty walking. It is inherited most commonly in an autosomal dominant fashion, but it can occur in an autosomal recessive fashion, although X-linked and sporadic cases have also been reported. This condition can present at any age from childhood to late adulthood; however, more frequently the onset is between the second and fourth decades of life. Clinically, it has been divided into pure and complicated forms. The pure form presents only with lower extremity spasticity, whereas sensory and other neurologic functions are intact. The complicated form has other neurologic features, including optic neuropathy, deafness, amyotrophy, peripheral neuropathy, ataxia, dementia, mental retardation, and extrapyramidal dysfunction. Several genetic mutations have been identified involving various HSP loci and genes. The most common mutation is on chromosome 2p22 involved with the SPAST gene encoding for the protein spastin, which is inherited in an autosomal dominant fashion. Other proteins involved include atlastin, paraplegin, spartin, and maspardin, among others. Treatment is symptomatic, with pharmacologic treatment of spasticity and supportive care for disability.
    This patient does not have Hirayama disease, amyotrophic lateral sclerosis (ALS), progressive muscular atrophy (PMA), or multiple sclerosis. Hirayama disease involves the upper extremities with predominant lower motor neuron findings (discussed in question 37). ALS manifests with a combination of upper and lower motor neuron findings. PMA (discussed in question 10) manifests with lower motor neuron findings, which are not seen in this patient. There are no features to support the diagnosis of multiple sclerosis on this patient.
    Bradley WG, Daroff RB, Fenichel GM, et al. Neurology in Clinical Practice, 5th ed. Philadelphia, PA: Elsevier; 2008.
    Ropper AH, Samuels MA. Adams and Victor’s Principles of Neurology, 9th ed. New York: McGraw-Hill; 2009.
How well did you know this?
1
Not at all
2
3
4
5
Perfectly
8
Q
  1. A 78-year-old man who has a history of diabetes and hypertension presents in septic shock with a blood pressure of 60/40 mm Hg. He is admitted to the ICU, where antibiotics and pressors are started. Over the next few days he improves; however, it is noticed that he cannot move his legs, and has a sensory level approximately at T6. His MRI of the spine shows a midthoracic spinal cord T2 hyperintensity. The most common mechanism for this patient’s condition is:
    a. Infarct in the territory of the artery of Adamkiewicz
    b. Watershed infarct
    c. Epidural abscess
    d. Aortic dissection
    e. Epidural hematoma
A
  1. b
    This patient has a watershed infarct of the spinal cord, which may occur after prolonged hypotension. The area of the spinal cord most susceptible to watershed infarcts is at the upper midthoracic levels (between T4 and T8), given that blood supply is scarce between the blood supply coming from the vertebral circulation and the aortic circulation, though watershed infarcts at lower levels have been described. Patients with vascular risk factors for atherosclerotic disease are at higher risk for developing this type of spinal cord infarct.
    The artery of Adamkiewicz is a large radicular artery that arises between T8 and L3 and supplies the lower thoracic and upper lumbar regions. This patient most likely has an infarct in the watershed region rather than in the area supplied by the artery of Adamkiewicz. The patient does not have a history to suggest an aortic dissection.
    In both epidural abscess and epidural hematomas, the MRI will show an epidural collection that was not present in this patient.
    Blumenfeld H. Neuronatomy through Clinical Cases, 1st ed. Sunderland, MA: Sinauer Associates; 2002.
    Ropper AH, Samuels MA. Adams and Victor’s Principles of Neurology, 9th ed. New York: McGraw-Hill; 2009.
How well did you know this?
1
Not at all
2
3
4
5
Perfectly
9
Q
  1. A 45-year-old woman presents with bilateral upper extremity weakness of 3 years’ duration. She began experiencing weakness of finger flexors and extensors in the right hand 2 years earlier, with similar weakness later occurring in the left hand. Over the prior year, the weakness had progressed and she could no longer abduct her arms beyond 30 degrees or fully flex or extend her forearms. Physical examination revealed significant atrophy of proximal upper extremity muscles, occasional fasciculations in the deltoids and biceps bilaterally, diminished tone in the arms, and Medical Research Council 2/5 strength in most muscle groups of the upper extremities. Left hip flexors were 4/5; otherwise motor power in the lower extremities was 5/5. Legs had normal tone, no ankle clonus, and absent Babinski sign bilaterally. Deep tendon reflexes could not be elicited in the upper extremities and were 1+ in the lower extremities. EMG showed evidence of widespread motor neuron dysfunction. What is the most likely diagnosis in this patient?
    a. Progressive muscular atrophy
    b. Amyotrophic lateral sclerosis
    c. Postpolio syndrome
    d. Adult-onset spinal muscular atrophy
    e. Inclusion body myositis
A
  1. a
    Progressive muscular atrophy (PMA) is considered along the spectrum of amyotrophic lateral sclerosis (ALS), although this is controversial; it is a motor neuron disease that affects only the lower motor neurons, distinguishing it from ALS, which affects both upper and lower motor neurons. PMA often presents with focal asymmetric distal weakness that later involves more proximal regions and other extremities, with lower motor neuron features on examination, such as atrophy, hyporeflexia, and fasciculations. PMA begins at an earlier age as compared to ALS, and survival is often longer than ALS, with a median survival of 5 years, though more rapidly progressive and more chronic forms of the disease occur. Bulbar and respiratory involvement occur later in the disease as compared to ALS. Laboratory evaluation may reveal moderately elevated creatine kinase, but never more than 10 times normal, and EMG shows evidence of motor neuron disease.
    Patients with ALS can present with predominantly lower motor neuron features early in their disease, with upper motor neuron findings not occurring until later. Therefore, a diagnosis of PMA is usually reserved for patients who have electrodiagnostic evidence of motor neuron disease and isolated lower motor neuron findings at least 3 years from symptom onset. The poliovirus infects anterior horn cells, leading to a lower motor neuron pattern of weakness similar to PMA. In those who survive, a postpolio syndrome may emerge years after recovery, marked by progressive fatigue, and weakness in muscles previously affected minimally or seemingly not at all. The history provided in this case is not consistent with postpolio syndrome. Unlike PMA, which progresses, on average, over 3 to 5 years, adult-onset spinal muscular atrophy is even more indolent and often affects predominantly proximal muscles. Inclusion body myositis is on the differential diagnosis of PMA and has a very indolent onset; relatively selective weakness of deep finger flexors and knee extensors in the setting of normal or only slightly elevated (less than two times normal) serum creatine kinase should prompt work-up for this disorder, including EMG and muscle biopsy.
    Bradley WG, Daroff RB, Fenichel GM, et al. Neurology in Clinical Practice, 5th ed. Philadelphia, PA: Elsevier; 2008.
How well did you know this?
1
Not at all
2
3
4
5
Perfectly
10
Q
  1. Regarding the vascular supply of the spinal cord, which of the following is incorrect?
    a. There is one anterior spinal artery that supplies the anterior two-thirds of the spinal cord
    b. There is one posterior spinal artery that supplies the posterior one-third of the spinal cord
    c. Segmental arteries arising from the aorta and internal iliac arteries feed the circulation at the thoracic and lumbar levels
    d. There is an epidural venous plexus system that connects pelvic venous plexuses and the intracranial venous system
    e. The anterior spinal artery originates from the vertebral arteries
A
  1. b
    There are two paired posterior spinal arteries.
    The blood supply to the spinal cord is provided by an arterial network that runs longitudinally along the cord, and its core is conformed by one anterior and two posterior spinal arteries. The anterior spinal artery suppli es the anterior two-thirds of the spinal cord and originates from the vertebral arteries just before they join to form the basilar artery. The two paired posterior spinal arteries supply the posterior third of the spinal cord and originate from the vertebral arteries as well. These three arteries receive contributions from multiple segmental and radicular arteries, which arise from intercostal and iliac arteries that originate from the aorta. The largest radicular artery is the artery of Adamkiewicz, which supplies the lower thoracic and upper lumbar regions of the spinal cord.
    The cervical and upper thoracic regions receive multiple collaterals from the vertebral arteries and other cervical vessels. Similarly, the conus medullaris and cauda equina are also richly vascularized from the contribution of multiple radicular arteries. There is a watershed region in between these two well-perfused regions located between T4 and T8. Between the anterior and posterior spinal circulations there is a circumferential network as well.
    There are anterior and posterior venous systems that drain into radicular veins and eventually into the epidural venous plexus system. This epidural venous plexus is a valveless system and extends from the pelvic region to the intracranial venous system. This could explain metastatic lesions in the CNS originating from the pelvic region.
    Blumenfeld H. Neuronatomy through Clinical Cases, 1st ed. Sunderland, MA: Sinauer Associates; 2002.
    Ropper AH, Samuels MA. Adams and Victor’s Principles of Neurology, 9th ed. New York: McGraw-Hill; 2009.
How well did you know this?
1
Not at all
2
3
4
5
Perfectly
11
Q
  1. A 69-year-old man who has hypertension, hyperlipidemia, diabetes, coronary disease, and peripheral vascular disease undergoes an endovascular intervention for a thoracoabdominal aneurysm. After the procedure he has new neurologic findings attributed to an anterior spinal artery infarct of the spinal cord. On examination you expect to find:
    a. Paraplegia with bilateral loss of sensation to pain and temperature below the lesion, and preserved sensation to vibration and proprioception
    b. Paraplegia with bilateral loss of sensation to vibration and proprioception below the lesion, and preserved sensation to pain and temperature
    c. Loss of sensation to vibration and proprioception bilaterally, and preserved sensation to pain and temperature, with no weakness
    d. Loss of sensation to pain and temperature bilaterally, and preserved sensation to vibration and proprioception, with no weakness
    e. Weakness on one side with loss of sensation to vibration and proprioception ipsilaterally, and loss of sensation to pain and temperature on the contralateral side
A
  1. a
    The spinal cord has ascending and descending pathways distributed in the white matter funiculi. The corticospinal tract originates from the primary motor cortex. Approximately 85% of the fibers decussate at the level of the pyramids and descend in the lateral funiculi as the lateral corticospinal tract. Fifteen percent of the fibers descend uncrossed as the anterior corticospinal tract.
    The dorsal columns of the spinal cord are formed by the fasciculi of gracilis and cuneatus, both carrying information related to vibration and proprioception. These fasciculi ascend ipsilaterally to the nucleus gracilis and cuneatus in the dorsal medulla. Fibers from these nuclei form the medial lemniscus, which decussates in the brain stem and ascends to the thalamus.
    Sensation of pain, temperature, and crude touch is carried by the lateral spinothalamic tract. Peripheral sensory fibers enter the cord through the dorsal rami and cross over two to three segments above the level of entry to the contralateral side, where they enter the lateral spinothalamic tract. They ascend to synapse in the thalamus.
    The anterior spinal artery provides blood supply to the anterior two-thirds of the spinal cord, perfusing the areas containing the corticospinal and spinothalamic tracts, but sparing the dorsal columns supplied by the posterior spinal arteries. Therefore, an anterior spinal artery infarct would manifest with paraplegia below the level of the infarct, as well as loss of sensation to pain and temperature, but spare the sensation to vibration and proprioception.
    Option C can be produced by an infarct in the posterior spinal arteries territory, which is uncommon. Option E is consistent with Brown-Séquard (hemisection) syndrome, with ipsilateral loss of motor function and sensation to vibration and proprioception below the level of the lesion, and contralateral loss of sensation to pain and temperature. This happens because the lesion affects the spinothalamic tract after its decussation, and the corticospinal tracts and dorsal columns before the decussation. Options B and D are not likely in the setting of this patient.
    Blumenfeld H. Neuronatomy through Clinical Cases, 1st ed. Sunderland, MA: Sinauer Associates; 2002.
    Ropper AH, Samuels MA. Adams and Victor’s Principles of Neurology, 9th ed. New York: McGraw-Hill; 2009.
How well did you know this?
1
Not at all
2
3
4
5
Perfectly
12
Q
  1. A 70-year-old man complains of back pain, lower extremity weakness, and sensory deficit to all modalities below his mid-abdominal region for the past 3 days. Since onset, his neurologic manifestations have progressed rapidly such that he is paraplegic on the day of evaluation. His MRI is shown in Figure 11.2. The most important risk factor for this patient’s condition is:

FIGURE 11.2 (A) Sagittal STIR MRI; (B) sagittal T2-weighted MRI

a. An episode of hypotension
b. Recent aortic manipulation
c. Atherosclerosis
d. The presence of a dural arteriovenous fistula
e. Use of warfarin for atrial fibrillation

A
  1. e
    Figure 11.2 shows an epidural collection consistent with an epidural hematoma. The most common initial symptom is back pain, followed by development of a myelopathic syndrome as the hematoma compresses the cord. It is more common in males and more frequent in the thoracolumbar region. Major risk factors for spinal epidural hematoma include anticoagulation either from medications or from coagulopathies and thrombocytopenia. Other factors that may increase the risk of this condition are trauma, neoplasms, pregnancy, and vascular malformations.
    Dural arteriovenous fistula is the most common vascular malformation of the spinal cord, and presents with an insidious and slowly progressive myelopathic syndrome, sometimes with acute exacerbations. These manifestations are caused by increased venous congestion and mass effect in the spinal cord, leading to venous infarcts. Acute hemorrhages may occur; however, dural arteriovenous fistulas rarely produce epidural hematomas. MRI of the spine may show cord signal abnormalities and flow voids, but the definitive diagnostic procedure is conventional angiography.
    Atherosclerosis, aortic dissection, and aortic manipulation (especially above the renal arteries) are associated with ischemic infarcts of the spinal cord. Prolonged hypotension may be associated with watershed infarcts of the spinal cord. Hypotension, atherosclerosis, and aortic manipulation are not typically associated with spinal epidural hematomas.
    Kreppel D, Antoniadis G, Seeling W. Spinal hematoma: A literature survey with meta-analysis of 613 patients. Neurosurg Rev. 2003; 26:1–49.
    Ropper AH, Samuels MA. Adams and Victor’s Principles of Neurology, 9th ed. New York: McGraw-Hill; 2009.
How well did you know this?
1
Not at all
2
3
4
5
Perfectly
13
Q
  1. Which of the following statements is incorrect regarding the pathophysiology of amyotrophic lateral sclerosis (ALS)?
    a. Glutamate-induced excitotoxicity has been postulated as a cause
    b. Mutations in the superoxide dismutase gene account for approximately 20% of familial ALS
    c. No environmental exposures have been definitively identified to be associated with ALS
    d. A rare form of ALS with parkinsonism and dementia had been identified in a geographically restricted area in the Western Pacific island of Guam
    e. The majority of ALS cases are familial and most often X-linked
A
  1. e
    The majority of amyotrophic lateral sclerosis (ALS) cases are sporadic. Familial ALS is usually autosomal dominant in inheritance, although autosomal recessive forms occur and X-linked inheritance is rare.
    The cause of ALS is unknown; there are various hypotheses and several environmental exposures postulated and investigated as causative but without definite evidence of associations. The majority of ALS cases are sporadic; approximately 10% are familial. Several gene mutations or deletions have now been identified, resulting in differing patterns of inheritance (most frequently autosomal dominant, but also autosomal recessive and rarely X-linked), age of onset, and phenotypic variability. Mutations in the copper/zinc superoxide dismutase (SOD1) gene on chromosome 21 account for approximately 20% of cases of familial ALS and less than 5% of sporadic ALS cases. The abnormal SOD1 protein usually maintains its dismutase function, but appears to damage motor neurons through other mechanisms, including a toxic gain of function of the mutant protein. More than 160 mutations have been identified, most often inherited in an autosomal dominant fashion. A rare form of ALS associated with parkinsonism and frontotemporal dementia was identified in a geographically restricted area in the Western Pacific island of Guam. Several theories abound regarding this rare syndrome, including exposure to cyanobacterial toxins and the presence of specific genetic mutations in the indigenous population.
    Bradley WG, Daroff RB, Fenichel GM, et al. Neurology in Clinical Practice, 5th ed. Philadelphia, PA: Elsevier; 2008.
How well did you know this?
1
Not at all
2
3
4
5
Perfectly
14
Q

15. A 69-year-old man presents with 6 months of gradually progressive lower extremity weakness. His MRI is shown in Figure 11.3. Regarding this patient’s condition, which of the following is correct?

FIGURE 11.3 Sagittal T2-weighted MRI

a. On examination there will be evidence of atrophy and fasciculations in the lower extremities
b. On examination there will be evidence of spinal shock
c. Steroids are indicated
d. Radiation therapy should be started immediately
e. On examination there will be spasticity in the lower extremities with hyperreflexia and upgoing toes

A
  1. e
    The patient’s history and MRI shown in Figure 11.3 are consistent with a cervical spondylotic myelopathy with spinal cord compression, which has occurred slowly over months. This condition is the most common cause of spinal cord compression in the elderly. Spondylosis is a degeneration of the spinal column, in which there is formation of osteophytes that eventually lead to compression of the spinal cord and nerve roots. Along with this, there are also disc herniations and ligamentum flavum hypertrophy, leading to narrowing of the spinal canal. Because the degenerative process progresses slowly, the neurologic manifestations develop insidiously, unlike acute cord compressions in which the patient has manifestations of spinal shock, which include weakness below the level of the lesion with flaccidity and hyporeflexia, sensory loss, and sphincteric dysfunction.
    Patients with cervical spondylotic myelopathy gradually develop neck stiffness and pain, weakness at and below the compression level, and unsteady gait. The examination shows findings of upper motor neuron signs below the level of compression, with spastic paraparesis, hyperreflexia, and upgoing toes. In the upper extremities there may be evidence of lower motor neuron signs, such as areflexia and atrophy, mainly at the same level of the compression. Patients have sensory deficits and may experience L’hermitte’s phenomenon, an electric sensation radiating down the back that occurs following neck flexion. The diagnostic test of choice is MRI of the spine. Goals of treatment include prevention of further neurologic deficits and therapies to help improve existing ones. Surgical decompression is the treatment of choice. Nonoperative options may provide pain relief; however, once myelopathic findings are evident surgical intervention may be required.
    Radiation therapy plays no role in the treatment of cervical spondylotic myelopathy, and is reserved for the treatment of radiosensitive neoplasms. Steroid therapy is not indicated and is mainly used in patients with traumatic spinal injury and in neoplastic cord compression.
    Ropper AH, Samuels MA. Adams and Victor’s Principles of Neurology, 9th ed. New York: McGraw-Hill; 2009.
How well did you know this?
1
Not at all
2
3
4
5
Perfectly
15
Q
  1. A 59-year-old man from Cape Town presents with progressive lower extremity weakness associated with numbness and paresthesias in both legs, as well as bladder and bowel incontinence. He has a history of diabetes, hypertension, chronic obstructive pulmonary disease with long-standing intermittent use of steroids, and prostate cancer with metastasis to the lower thoracic spine, treated with radiation 12 months prior. His MRI shows T2 hyperintensity in the thoracic spinal cord, with no evidence of compression or extradural mass. What is the most likely diagnosis?
    a. Lathyrism
    b. Transient radiation myelopathy
    c. Epidural lipomatosis
    d. Delayed radiation myelopathy
    e. Konzo
A
  1. d
    This patient has had radiation to the spine with development of radiation-induced myelopathy. There are two types of radiation-induced myelopathy, a transient and a delayed form. The transient form happens early, approximately 3 to 6 months after the radiation treatment, and presents with dysesthesias in the extremities that eventually resolve without sequelae. The delayed form occurs 6 months or greater following radiation therapy, as in this case. The presentation is insidious with paresthesias and dysesthesias of the feet, L’hermitte’s phenomenon, and progressive weakness of the legs. Eventually, bowel and bladder can be affected. The MRI shows increased T2 signal in the affected regions, sometimes with heterogeneous gadolinium enhancement. Steroids have been tried; however, the key is prevention by minimizing the dose of radiation used.
    Lathyrism is a neurotoxic disorder presenting as a myelopathy with subacute spastic paraparesis. It is endemic in certain parts of Ethiopia, India, and Bangladesh, and occurs from consumption of Lathyrus sativus, a legume (grass pea or chick pea) that contains the toxin β-N-oxalylamino-l-alanine. It is more prevalent in poor populations.
    Konzo is a type of myelopathy that presents with a spastic paraparesis of abrupt onset, sometimes associated with involvement of the visual pathways. It is most commonly seen in certain parts of Africa and results from consumption of poorly processed Cassava, which contains cyanide. It is more prevalent in droughts and in poor populations.
    Epidural lipomatosis is a condition in which there is hypertrophy of extramedullary adipose tissue in the epidural space, and is usually associated with chronic use of steroids. Patients present with back pain and myelopathic findings. The treatment involves stopping steroids, and sometimes even surgical decompression. MRI of the spine demonstrates the fatty tissue within the spinal canal producing spinal stenosis, which was not described in this patient.
    Bradley WG, Daroff RB, Fenichel GM, et al. Neurology in Clinical Practice, 5th ed. Philadelphia, PA: Elsevier; 2008.
    Ropper AH, Samuels MA. Adams and Victor’s Principles of Neurology, 9th ed. New York: McGraw-Hill; 2009.
How well did you know this?
1
Not at all
2
3
4
5
Perfectly
16
Q
  1. A 45-year-old woman is evaluated for progressive spastic paraparesis. She also has anemia and leukopenia. Her vitamin B12 level is 300 pg/mL (220 to 700 pg/mL), folate level is 1.9 mg/mL (2 to 18 mg/mL), and copper is 90 μg/dL (85 to 155 μg/dL). Which of the following is correct in the treatment of this patient?
    a. Start copper supplementation and observe for improvement before providing vitamin B12 or folate
    b. Start folate supplementation
    c. Check methylmalonic acid and homocysteine, and start vitamin B12 and folate supplementation
    d. Start zinc supplementation
    e. No therapies are needed because the test results are within normal limits
A
  1. c
    This patient likely has a myelopathy and hematologic disturbances from a metabolic cause. Deficiency of vitamin B12 and/or copper can lead to this clinical picture and it is difficult to differentiate the etiology; therefore, blood levels of these vitamins and trace elements should be checked. Methylcobalamine acts as a cofactor for methionine synthase in the reaction where homocysteine is converted into methionine. This enzyme also requires folate as a cosubstrate. This metabolic pathway is important for DNA synthesis, and when there is lack of the cofactors or substrates, there is dysfunction leading to the clinical manifestations including myelopathy and hematologic disturbances.
    Folate deficiency can lead to neurologic disturbances like those seen in vitamin B12 deficiency; however, they are less frequent and less severe. When patients have vitamin B12 and folate deficiency, and only folate is supplemented, hematologic abnormalities improve, but not the neurologic manifestations. Therefore, both should be supplemented at the same time. In this case, folate levels are low but vitamin B12 levels are within the normal range. However serum vitamin B12 levels can be normal in some patients with vitamin B12 deficiency, and serum methylmalonic acid (MMA) and total homocysteine levels should be checked because they are more sensitive in detection of these deficiencies.
    Patients with copper deficiency can also have these clinical manifestations; however, this deficiency is encountered less frequently than that of vitamin B12 and folate. In this case, the copper level is borderline low and should be rechecked in the future; this level would in all likelihood not explain all the manifestations in this patient.
    Zinc induces the synthesis of metallothionein in enterocytes. Copper has high affinity for metallothionein and will bind to it, entering the enterocytes that eventually are sloughed off the mucosa, leading to loss of copper. Therefore, zinc ingestion may be associated with copper deficiency and may decrease copper levels.
    In this patient, MMA and homocysteine should be checked, and vitamin B12 along with folate should be started.
    Bradley WG, Daroff RB, Fenichel GM, et al. Neurology in Clinical Practice, 5th ed. Philadelphia, PA: Elsevier; 2008.
    Ropper AH, Samuels MA. Adams and Victor’s Principles of Neurology, 9th ed. New York: McGraw-Hill; 2009.
How well did you know this?
1
Not at all
2
3
4
5
Perfectly
17
Q
  1. A 33-year-old man from Cleveland with unknown past medical history presents to the hospital in July with fever, mild lethargy, and bilateral lower extremity weakness, which has progressed over the past 5 days. On examination he is mildly confused with flaccid quadriparesis and areflexia. Which of the following is the most likely etiology?
    a. An enterovirus
    b. A flavivirus

c. HTLV-1
d. HTLV-2
e. A herpesvirus

A
  1. b
    This patient has West Nile virus (WNV) infection. WNV is a flavivirus transmitted by mosquitoes. It causes an illness characterized by meningitis, encephalitis, and myeloradiculitis. Cases in the United States present during summer months, and the initial symptom is fever, progressing to altered mental status, gastointestinal symptoms, back pain, and flaccid weakness with areflexia, more often proximal and asymmetric and sometimes affecting all limbs. The progression to nadir occurs in 3 to 8 days.
    The diagnosis is made with serology, CSF antibodies, and/or PCR. CSF shows neutrophilic pleocytosis, with high protein and normal glucose. MRI shows cauda equina, spinal cord, and/or leptomeningeal enhancement. Pathologic studies have shown perivascular inflammation with anterior horn cell loss in the spinal cord.
    Many enteroviruses and herpesviruses are associated with a transverse myelitis, usually as a post-infectious phenomenon; there are no features of transverse myelitis in this case. Poliovirus is an enterovirus that can lead to areflexic flaccid paralysis from damage to anterior horn cells; however, poliovirus does not cause the encephalopathic changes seen in WNV, and furthermore, it has been eradicated from the United States.
    This patient does not have a history to suggest HIV, which causes a gradually progressive myelopathy characterized by spastic paraparesis with impaired vibration and proprioception sensation, and sensory gait ataxia. The pathologic finding in HIV myelopathy is microvacuolar changes. Myelopathy from HIV is usually a late disease manifestation, and does not seem to be the case in this patient.
    HTLV-1 is a virus transmitted via sexual, parenteral, or maternal routes, which causes a chronic progressive myelopathy known as tropical spastic paraparesis. The myelopathy is usually localized in the thoracic region, and pathologically, there is neuronal cell loss, microvacuolization, and long tract degeneration. The diagnosis is made by detecting antibodies to HTLV-1 in the serum and CSF, as well as PCR. This virus is also associated with adult T-cell lymphoma and leukemia. HTLV-2 is also associated with a progressive myelopathy and can be seen in native Americans, intravenous drug users, and patients with HIV, but this is much less common than HTLV-1.
    Jeha LE, Sila CA, Lederman RJ, et al. West Nile virus infection: A new acute paralytic illness. Neurology. 2003; 61:55–59.
    Ropper AH, Samuels MA. Adams and Victor’s Principles of Neurology, 9th ed. New York: McGraw-Hill; 2009.
How well did you know this?
1
Not at all
2
3
4
5
Perfectly
18
Q
  1. A previously healthy 36-year-old woman presents with rapidly progressive flaccid paraparesis. She recalls an episode of right-eye visual loss about 2 years ago, which resolved without treatment. Her spine MRI shows a large T2-hyperintense lesion extending from T5 to T11. MRI of the brain shows few scattered nonspecific white matter lesions. Her ESR is elevated and antinuclear antibody (ANA) is positive. CSF analysis shows 49 WBCs/mm3 (normal is up to 5 lymphocytes/mm3), half of which are polymorphonuclear leucocytes, a protein level of 92 mg/dL (normal is up to 45 mg/dL), normal immunoglobulin G (IgG) index, and no oligoclonal bands. Which of the following is incorrect regarding this condition?
    a. In neuromyelitis optica, ANA positivity suggests a separate diagnosis of systemic lupus erythematosus
    b. Neuromyelitis optica–IgG is an antibody against aquaporin-4
    c. Asians and African populations are more frequently affected
    d. Pathologic analysis of the spinal cord tissue will show inflammation, demyelination, and necrosis
    e. Antibodies against aquaporin-4 are part of the diagnostic criteria
A
  1. a
    This patient has neuromyelitis optica (NMO) or Devic’s disease, which is an inflammatory demyelinating disease of the CNS affecting the optic nerves and spinal cord. It is nine times more common in women than in men, and unlike multiple sclerosis, it tends to affect more Asian and African populations. Patients present with a combination of optic neuritis and myelitis. The myelitis usually presents acutely as a longitudinally extensive transverse myelitis extending over three or more segments. This commonly causes bilateral signs and symptoms, which can be severe.
    MRI of the spinal cord demonstrates T2 hyperintensity and gadolinium enhancement. MRI of the brain may show white matter changes; however, these changes are nonspecific and not necessarily diagnostic for multiple sclerosis. There are sometimes unusual brain changes in areas where aquaporin channels are common, such as in the brain stem and hypothalamus. Occasionally posterior reversible encephalopathy syndrome changes will be present.
    CSF is frequently abnormal, showing increased WBCs and increased protein with normal glucose. Oligoclonal bands are rarely present in the CSF, in contrast to multiple sclerosis. Pathologic analysis of the spinal cord tissue will show inflammation and demyelination, with polymorphonuclear infiltrates and eosinophils, associated with necrosis.
    The NMO-IgG antibody helps make the diagnosis and is part of the diagnostic criteria. It has a sensitivity of 64% and a specificity of 99%. This antibody is directed against aquaporin-4, which is a protein found in astrocyte foot processes around cerebral microvessels located at the blood-brain barrier.
    A majority of patients will have nonspecific seropositivity to other autoantibodies, including the antinuclear antibody. The presence of these antibodies does not establish the presence of other autoimmune disease nor does it establish causality. It is possible that these seropositivities are a result of a general predisposition to autoimmunity.
    Matiello M, Jacob A, Wingerchuk DM, et al. Neuromyelitis optica. Curr Opin Neurol. 2007; 20:255–260.
    Wingerchuk DM, Lennon VA, Pittock SJ, et al. Revised diagnostic criteria for neuromyelitis optica. Neurology. 2006; 66:1485–1489.
How well did you know this?
1
Not at all
2
3
4
5
Perfectly
19
Q
  1. A 34-year-old man has progressive neurologic manifestations. He was a normal child and adolescent, and he was fine until he was about 23 years of age when he started noticing numbness and tingling in his feet and difficulty running. He has been diagnosed with a spastic paraparesis and has been getting gradually worse over the past few years to the point that now he needs a walker to walk. He also has very mild cognitive impairment, behavioral problems, mild hearing and visual impairment, as well as urinary incontinence. His past medical history is significant for at least five admissions to the ICU secondary to upper respiratory infections, leading to significant hypotension and hypoglycemia, for which he has required steroids. Regarding this condition, which of the following is correct?
    a. He has amyotrophic lateral sclerosis
    b. He has pure hereditary spastic paraparesis
    c. The most likely diagnosis is neuromyelitis optica
    d. He has adrenomyeloneuropathy
    e. He has primary lateral sclerosis
A
  1. d
    This patient has adrenomyeloneuropathy, which is one of the phenotypes of adrenoleukodystrophy. This is a peroxisomal disorder transmitted in an X-linked fashion and associated with a mutation in the ABCD1 gene on chromosome Xq28, which encodes a peroxisomal adenosine triphosphate-binding cassette transporter protein. Because of this mutation there is an impaired ability to oxidize very long chain fatty acids (VLCFAs), especially hexacosanoic acid, leading to the accumulation of VLCFAs in tissues and plasma.
    In adrenoleukodystrophy there are four main phenotypes; an early onset cerebral white matter disease (adrenoleukodystrophy), adrenomyeloneuropathy, isolated Addison’s disease, or asymptomatic. In adrenomyeloneuropathy, male patients begin having manifestations around age 20, with slowly progressive paraparesis, sensory neuropathy, problems with sphincter control, mild hypogonadism, and mild cognitive impairment. Some patients may develop hearing and visual impairment as well. Most patients also develop adrenal insufficiency. Increased levels of VLCFAs in plasma and cultured fibroblasts help in the diagnosis. Patients require steroids for the adrenal insufficiency; however, this medication does not have an effect on the CNS involvement. Very early bone marrow transplantation may be a therapeutic option.
    Neither amyotrophic lateral sclerosis, primary lateral sclerosis, hereditary spastic paraparesis, nor neuromyelitis optica lead to adrenal insufficiency or the cognitive, behavioral, or sensory features that the patient in the case has.
    Fenichel GM. Clinical Pediatric Neurology: A Signs and Symptoms Approach, 6th ed. Philadelphia, PA: Saunders Elsevier; 2009.
How well did you know this?
1
Not at all
2
3
4
5
Perfectly
20
Q
  1. A patient presents with a several months’ history of gradually progressive sensory loss to pain and temperature over his shoulders and both arms bilaterally, with preserved sensation to light touch, vibration, and proprioception. On examination he has findings suggestive of multiple old injuries in both upper limbs. He also has weakness and atrophy in both upper extremities, with minimal findings in the lower extremities. Which of the following is correct regarding this condition?
    a. Syringomyelia can cause this syndrome
    b. MRI will likely show an infarct in the anterior spinal artery territory
    c. The syndrome is consistent with Brown-Séquard syndrome
    d. The findings suggest subacute combined degeneration of the spinal cord
    e. This patient has a watershed infarct of the spinal cord
A
  1. a
    This patient has a dissociated sensory loss with loss of sensation to pain and temperature and preserved sensation to light touch, vibration, and proprioception in his upper extremities. This clinical syndrome is seen with central spinal cord lesions in which there is compromise of crossing fibers in the midline anterior to the central canal, which carry sensory input related to pain and temperature. Usually the distribution of this sensory loss is described as “cape-like” or “shawl-like”. Because it does not affect the posterior columns, the sensory modalities carried by this pathway are not affected. This dissociated sensory loss is seen in syringomyelia, in which there is a cavitation in the central parts of the spinal cord, usually in the cervical region, sometimes extending upward to the brain stem or downward to the thoracic region. It has been described that patients with syringomyelia, given their lack of sensory input in their upper extremities, may experience repeated trauma and recurrent injuries. Syringomyelia may be associated with other developmental anomalies of the vertebral column or skull, as well as with Chiari malformations.
    This patient does not have a spinal cord infarct, because the history is that of a gradually progressive illness, and there is no significant motor disturbance, such as that seen from infarcts affecting the anterior horns and corticospinal tracts.
    Brown-Séquard syndrome, or hemisection of the spinal cord, is a characteristic syndrome in which there is loss of pain and temperature sensation contralateral to the side of the lesion due to interruption of the crossed spinothalamic tract. There is also ipsilateral loss of proprioception and vibration sensation below the level of the lesion from interruption of the ipsilateral posterior columns, as well as ipsilateral weakness below the lesion from corticospinal tract involvement.
    Blumenfeld H. Neuronatomy through Clinical Cases, 1st ed. Sunderland, MA: Sinauer Associates; 2002.
    Ropper AH, Samuels MA. Adams and Victor’s Principles of Neurology, 9th ed. New York: McGraw-Hill; 2009.
How well did you know this?
1
Not at all
2
3
4
5
Perfectly
21
Q
  1. Which of the following is correct regarding neoplastic disease of the spinal cord?
    a. Ependymomas are the most common intramedullary spinal cord tumor in children

b. Astrocytomas are the most common intramedullary spinal cord tumor in adults
c. Myxopapillary ependymomas originate from the filum terminale
d. Meningioma is the most common extradural tumor of the spinal cord
e. Intramedullary spinal cord metastases are more common than primary intramedullary neoplasms

A
  1. c
    Neoplasms of the spinal cord can be divided into primary or metastatic. Also they can be divided anatomically into extradural, intradural extramedullary, and intradural intramedullary.
    Primary neoplasms accounting for intramedullary spinal cord metastases include small cell lung cancer, breast cancer, renal cell cancer, lymphoma, and melanoma. However, metastatic intramedullary disease is not very common, and primary intramedullary tumors are more frequent.
    Meningioma is a common intradural extramedullary tumor, which has predilection for the thoracic spine, and with its growth can produce spinal cord compression. Nerve sheath tumors such as schwannomas and neurofibromas are also intradural and extramedullary.
    The most frequent intradural intramedullary tumors found include ependymoma and astrocytoma. Astrocytomas are most common in children and are usually slow-growing low-grade tumors. Ependymomas are the most common intramedullary tumors in adults, usually arising from the central canal and expanding outward. Myxopapillary ependymoma is a type of ependymoma that arises from the ependymal cells in the filum terminale, and is the most common tumor in this location.
    Fenichel GM. Clinical Pediatric Neurology: A Signs and Symptoms Approach, 6th ed. Philadelphia, PA: Saunders Elsevier; 2009.
    Ropper AH, Samuels MA. Adams and Victor’s Principles of Neurology, 9th ed. New York: McGraw-Hill; 2009.
How well did you know this?
1
Not at all
2
3
4
5
Perfectly
22
Q
  1. A 12-year-old girl with no past medical history except a febrile illness about 1 week ago, presents with 3 days of headache, altered mental status, inability to move her legs, and urinary incontinence. On examination she is confused, has no meningeal signs, and has a flaccid paraplegia with areflexia and sensory loss below her waist. Her CSF shows 40 WBCs/mm3 (normal up to 5 lymphocytes/mm3) with lymphocytic predominance, a protein level of 70 mg/dL (normal up to 45 mg/dL) with normal glucose, and high immunoglobulin G index. MRI of the brain shows symmetric subcortical white matter lesions, and MRI of the spine shows an expanding intramedullary lesion between T5 and T10 with gadolinium enhancement. The most likely diagnosis isa. Multiple sclerosis
    b. Acute disseminated encephalomyelitis
    c. Neuromyelitis optica
    d. Idiopathic transverse myelitis
    e. Bacterial meningitis
A
  1. b
    This patient has an acute neurologic disorder temporally associated with a recent febrile illness, with findings suggestive of an inflammatory myelopathy and encephalopathy. At the time of presentation, the illness was apparently monophasic, with symmetric involvement of cerebral white matter and simultaneous spinal cord involvement. CSF findings support an inflammatory cause. In this case the most likely explanation for her clinical picture is acute disseminated encephalomyelitis (ADEM).
    ADEM is an inflammatory demyelinating disorder of childhood in which there is a monophasic immunologic reaction to a viral illness. Patients have an encephalopathy, with confluent white matter changes on MRI and inflammatory markers in the CSF. Occasionally, the spinal cord is also affected with features of a transverse myelitis, in which the lesion affects more than three segments of the spinal cord. The treatment is intravenous steroids in high doses.
    Encephalopathy is not a feature of neuromyelitis optica; the patient may have some white matter lesions, but in contrast to ADEM, they are not symmetric or confluent.
    Multiple sclerosis is an inflammatory demyelinating disorder that presents with a relapsing and remitting or a progressive course. When it affects the spinal cord, the lesions are usually in short segments, not like in this case. Acute encephalopathy is not common in multiple sclerosis.
    The diagnosis of idiopathic transverse myelitis is made when the patient has a transverse myelitis with no other explanation, and usually without other features. This patient has encephalopathy and white matter brain lesions, making transverse myelitis unlikely.
    The clinical, imaging, and CSF findings do not correlate with bacterial meningitis.
    Fenichel GM. Clinical Pediatric Neurology: A Signs and Symptoms Approach, 6th ed. Philadelphia, PA: Saunders Elsevier; 2009.
    Ropper AH, Samuels MA. Adams and Victor’s Principles of Neurology, 9th ed. New York: McGraw-Hill; 2009.
How well did you know this?
1
Not at all
2
3
4
5
Perfectly
23
Q
  1. A 50-year-old man presents with gradually progressive weakness in his upper extremities over the past 6 months. A cervical MRI is obtained, which is shown in Figure 11.4. On the basis of the location of the lesion, on examination you will find:

a. Patellar and ankle areflexia
b. Sensory level below the level of the nipples
c. Sparing of superficial abdominal reflexes
d. Horner’s syndrome
e. Flaccid weakness of his lower extremities

A
  1. d
    Horner’s syndrome is seen in patients with spinal cord lesions above T1, where the spinal sympathetic tract synapses before exiting the spinal cord (discussed in Chapters 1 and 10). This patient has a lesion above T1, and will likely have a Horner’s syndrome.
    As shown in Figure 11.4, this patient has an intramedullary lesion producing signal abnormality in the cervical cord from C1 to C6. Because the lesion has developed gradually, the patient will likely have myelopathic findings with upper motor neuron manifestations below the lesion. Therefore, the patient will likely have increased tone with spasticity and hyperreflexia below the level of the lesion. Some cases will demonstrate lower motor neuron findings at the same level of the lesion due to involvement of anterior horn cells, in this case probably in the upper extremities. A sensory level is helpful for clinical localization of the level of the lesion. The level of the nipple line correlates with T4. Other levels that are useful landmarks are the base of the neck (C4), umbilicus (T10), groin (L1), and anal region (S5). In this case the lesion is in the cervical region; therefore, the patient’s sensory level is likely above the nipple line. The presence of superficial abdominal reflexes is a normal finding, and their absence indicates a corticospinal tract lesion above the T6 segment.
    Blumenfeld H. Neuronatomy through Clinical Cases, 1st ed. Sunderland, MA: Sinauer Associates; 2002.
How well did you know this?
1
Not at all
2
3
4
5
Perfectly
24
Q
  1. All of the following conditions are associated with atlantoaxial dislocation, except:
    a. Rheumatoid arthritis
    b. Klippel-Feil syndrome
    c. Systemic lupus erythematosus
    d. Down’s syndrome
    e. Morquio syndrome
A
  1. c
    The ligaments between the atlas and the axis, as well as the odontoid process, are important in maintaining the stability of this articulation. When this articulation fails, or the odontoid is absent or damaged, atlantoaxial dislocation can occur, leading to neck pain and other manifestations of spinal cord compression including quadriplegia and death.
    Rheumatoid arthritis is known to produce a destructive inflammatory process of the ligaments that attach the odontoid process to the atlas and the skull, therefore causing atlantoaxial dislocation.
    Other causes of this condition include Klippel-Feil syndrome, Down’s syndrome, and Morquio syndrome. Klippel-Feil syndrome is a condition in which there is decreased number and abnormal fusion of cervical vertebrae. Morquio syndrome is a skeletal disease in which the odontoid process may be aplastic or absent, leading to secondary spinal cord disease.
    Systemic lupus erythematosus has been associated with syndromes consistent with myelitis or myelopathy, but not atlantoaxial dislocation.
    Fenichel GM. Clinical Pediatric Neurology: A Signs and Symptoms Approach, 6th ed. Philadelphia, PA: Saunders Elsevier; 2009.
    Ropper AH, Samuels MA. Adams and Victor’s Principles of Neurology, 9th ed. New York: McGraw-Hill; 2009
How well did you know this?
1
Not at all
2
3
4
5
Perfectly
25
Q
  1. Regarding neoplastic spinal cord disease, which of the following is incorrect?
    a. Malignancies that may metastasize to the spine include those arising from the breast, lung, prostate, and kidney
    b. Radiation therapy can improve pain and neurologic symptoms in patients with lymphoproliferative neoplasia producing cord compression
    c. High-dose 24-hour treatment of continuous IV methylprednisolone is the treatment of choice
    d. Intradural extramedullary tumors causing cord compression include neurofibromas, schwannomas, and meningiomas
    e. Intramedullary tumors include ependymomas and astrocytomas
A
  1. c
    Neoplastic disease of the spinal cord can be intrinsic to the cord or metastatic disease. Tumors affecting the spinal column and spinal cord can be anatomically divided into extradural or intradural, the latter further classified into intramedullary or extramedullary. All of these tumors can also be divided into metastatic or nonmetastatic disease. The most common nonmetastatic extramedullary tumors include neurofibromas, schwannomas, and meningiomas. The most frequent nonmetastatic intramedullary tumors are spinal cord astrocytomas and ependymomas.
    Metastatic disease to the spinal cord most commonly originates from breast, lung, prostate, and kidney cancer, although it is not limited to these. Other invading neoplastic lesions include lymphoma and multiple myeloma.
    Spinal cord neoplastic disease usually presents with pain and focal neurologic manifestations resulting from spinal cord compression or dysfunction. Patients have findings consistent with myelopathy, along with other manifestations of the underlying cancer.
    The diagnostic test of choice to determine the presence of neoplastic disease is MRI of the spinal cord with gadolinium.
    There are multiple treatment options that are selected depending on the underlying neoplasm, degree of neurologic disease, timing of presentation, functional status, and prognosis. Steroids are frequently used, especially in the acute phase of cord compression, with dexamethasone being the most commonly used. Methylprednisolone with a 30 mg/kg bolus and an infusion of 5.4 mg/kg/hour for 23 hours is a formulation used more commonly in spinal cord traumatic injury, but not for neoplastic disease.
    Radiation therapy is used for radiosensitive neoplasias, such as lymphoproliferative disease and germ cell tumors, and for various types of metastases, particularly in the setting of cord compression. It may be used as a palliative measure to improve symptoms of pain and other symptoms of cord compression.
    Surgical resection should be entertained when feasible, even in cases of metastatic disease, where it has shown superiority when combined with radiotherapy as compared to radiotherapy alone. However surgical treatments may not be possible in some neoplasms, such as in intramedullary lesions.
    Bradley WG, Daroff RB, Fenichel GM, et al. Neurology in Clinical Practice, 5th ed. Philadelphia, PA: Elsevier; 2008.
    Ecker RD, Endo T, Wetjen NM, et al. Diagnosis and treatment of vertebral column metastases. Mayo Clinic Proc. 2005; 80:1177–1186.
    Ropper AH, Samuels MA. Adams and Victor’s Principles of Neurology, 9th ed. New York: McGraw-Hill; 2009.
How well did you know this?
1
Not at all
2
3
4
5
Perfectly
26
Q
  1. A 27-year-old man is involved in a motor vehicle accident and is ejected from the car. He suffers acute spinal cord injury with cord transection at the level of C5 to C6. The following findings would be present during the first week of injury, except:
    a. Flaccid weakness of his lower extremities
    b. Atonic bladder with overflow incontinence
    c. Decreased rectal tone
    d. Patellar and ankle areflexia
    e. Spontaneous reflex defecation
A
  1. e
    Spinal cord trauma is a significant cause of myelopathy and common cause of severe disability after trauma, especially in young adults. Trauma produces spinal cord injury by multiple mechanisms, including direct transection of the cord, displacement of spinal column structures, disruption of axons, interruption of blood flow to the cord, hemorrhage, inflammation, and/or edema.
    After spinal cord injury there are two phases: an initial phase of spinal shock followed by a second phase of increased reflex activity with spasticity. The acute spinal shock phase occurs soon after the injury and lasts for a few weeks (between 1 and 6, but with no exact limits). It is manifested by suppression of spinal segmental activity below the level of the lesion. With complete spinal cord injury, the patient has a flaccid weakness with areflexia below the lesion, atonic bladder with overflow incontinence, distention of the bowel with absence of peristalsis and constipation, depressed rectal tone, and abolished genital reflexes. There is autonomic dysfunction below the lesion explaining the labile blood pressure seen in these cases.
    A few weeks after spinal cord injury, the patient will start presenting features of increased reflexic activity below the lesion. There is no clear chronologic boundary between this stage and the previous one of spinal shock. In this second stage, there are upper motor neuron findings with increased tone, leading to spasticity and hyperreflexia. Plantar responses are upgoing; triple flexion response commonly occurs. Patients will have detrusor spasms and hyperactivity with urinary loss secondary to spontaneous bladder contraction as well as spontaneous reflex defecation. These bladder and bowel reflexes are probably explained by the lack of inhibition of the sacral neurons by rostral centers, leading to spontaneous contraction secondary to a local reflex arc. Neither spontaneous reflex defecation nor detrusor contraction are present in the spinal shock phase.
    Ropper AH, Samuels MA. Adams and Victor’s Principles of Neurology, 9th ed. New York: McGraw-Hill; 2009.
27
Q
  1. A 55-year-old man with slowly progressive gait ataxia, myelopathic findings, and mild cognitive impairment presents for follow-up. He has a diagnosis of superficial siderosis of the CNS. Regarding this condition, which of the following is incorrect?
    a. There is hemosiderin deposition in the subpial layers of the brain and spinal cord
    b. CSF analysis shows xanthochromia
    c. Sensorineural hearing loss is a frequent manifestation
    d. The source of hemorrhage is commonly not identifiable
    e. MRI shows a T2-hyperintense rim around the brain, brain stem, and spinal cord
A
  1. e
    Superficial siderosis of the CNS is the result of hemosiderin deposition in the subpial layers of the brain and spinal cord, which is secondary to recurrent bleeding into the subarachnoid space. The bleeding source is undetermined in most cases. Some risk factors have been reported, such as previous subarachnoid hemorrhage, head trauma, and previous intradural surgery; however, the significance of these historical aspects is not clear.
    The most common manifestations include gait ataxia, dysarthria, and sensorineural hearing loss. Other clinical manifestations include anosmia, cognitive decline, and myelopathic findings. Xanthochromia and/or the presence of red blood cells is a common CSF finding. MRI shows T2 hypointensity around the brain, brain stem, and spinal cord, and sometimes intraspinal fluid collections are detected.
    If a possible etiology for the recurrent bleeding is found, endovascular or surgical repair may be entertained; however, it is unclear if this approach has a significant impact on progression of the disease.
    Kumar N, Cohen-Gadol AA, Wright RA, et al. Superficial siderosis. Neurology. 2006; 66:1144–1152.
28
Q
  1. A 52-year-old patient with HIV on no antiretroviral treatment comes with a history of long-standing pain in his legs, spasticity in his lower extremities, and gait ataxia. Studies obtained show a positive serum rapid plasma reagin (RPR). Regarding spinal cord disease in syphilis, which of the following is incorrect?
    a. Syphilis does not present as a spinal cord infarction
    b. The treatment for neurosyphilis is penicillin G 24 million units/day intravenously for 14 days
    c. Syphilis can present as transverse myelitis
    d. Patients with tabes dorsalis have prominent sensory ataxia and proprioceptive loss
    e. Other presentations of syphilis affecting the spinal cord include pachymeningitis and gummas in the spinal cord
A
  1. a
    Syphilis is caused by the spirochete Treponema pallidum and is associated with a wide variety of neurologic manifestations. Myelopathy is frequently encountered and can be produced by various mechanisms, including spinal cord infarction. Tabes dorsalis is the classic myelopathic syndrome described in neurosyphilis.
    Tabes dorsalis is characterized by dysfunction of the posterior columns with prominent sensory ataxia and proprioceptive loss, along with other manifestations such as sphincter dysfunction, L’hermitte’s sign, lancinating pain, and areflexia.
    The natural history of tabes dorsalis has been described in three different phases:
    * The preataxic phase in which the patient has lancinating pain of his legs, along with sphincter and sexual dysfunction
    * The ataxic phase with prominent proprioceptive loss, sensory ataxia leading to the development of Charcot joints
    * The postataxic or paralytic phase characterized by spastic paraparesis, autonomic dysfunction, worsening of the pain and sphincter dysfunction, and cachexia

Pathologic findings demonstrate inflammation, demyelination, and gliosis of the posterior columns and dorsal roots of the spinal cord.
Syphilis can also produce spinal cord disease by other mechanisms, such as spinal cord infarction from vasculitis, meningomyelitis manifested as an inflammatory transverse myelitis, hypertrophic pachymeningitis, and gummas, which can be intramedullary or extramedullary.
The treatment of neurosyphilis is penicillin G intravenously, with a dose of 24 million units daily in divided doses for 14 days.
Ropper AH, Samuels MA. Adams and Victor’s Principles of Neurology, 9th ed. New York: McGraw-Hill; 2009.

29
Q
  1. A 46-year-old woman with a history of pelvic cancer presents with severe radicular pain in the perineum radiating to both legs, right more than left. On examination she has asymmetric sensory loss in the lower extremities and perineal region, bilateral asymmetric lower extremity weakness, and areflexia. Which of the following is correct regarding this patient’s condition?
    a. She has a cauda equina syndrome
    b. She has a lesion in the conus medullaris
    c. She has an intramedullary tumor
    d. She has transverse myelitis
    e. She has an epidural hematoma
A
  1. a
    The conus medullaris is at the lower end of the spinal cord, beyond which the lumbar and sacral spinal nerve roots descend within the spinal canal as a bundle of fibers, called the cauda equina (meaning horse’s tail), to reach their corresponding exit foramina. Lesions in the lumbar or lumbosacral region can compress either the conus medullaris and/or the cauda equina, producing a distinctive clinical syndrome.
    A lesion affecting primarily the conus medullaris presents with perineal sensory deficit in a saddle distribution, which is usually bilateral and symmetric. Pain is often symmetric, but is not typically radicular. There is usually lower extremity symmetric weakness and sometimes decreased or absent ankle reflexes, but this may be mild. Bowel and bladder dysfunction occur early in the course of the disease, as well as sexual dysfunction.
    A lesion that affects the cauda equina presents with distinctive radicular pain and sensory loss with an asymmetric distribution. Motor deficits are also asymmetric with evidence of hyporeflexia. Bowel and bladder function may be affected, but usually this occurs later in the course and less frequently than with conus lesions. Spasticity and other upper motor neuron signs will not be present as this is a lower motor neuron disorder.
    This patient has features of cauda equina syndrome. This condition is produced by compression or irritation of the fibers, with causes including disc herniations, tumors, hematomas, or trauma, as well as infectious and inflammatory disorders (e.g., tuberculosis, cytomegalovirus, and sarcoidosis). There are no clinical features to suggest an intramedullary tumor, transverse myelitis, or an epidural hematoma.
    Brazis PW, Masdeu JC, Biller J. Localization in Clinical Neurology, 5th ed. Philadelphia, PA: Lippincott Williams & Wilkins; 2007.
    Ropper AH, Samuels MA. Adams and Victor’s Principles of Neurology, 9th ed. New York: McGraw-Hill; 2009.
30
Q
  1. A 56-year-old woman is referred to the clinic for evaluation of suspected amyotrophic lateral sclerosis (ALS). Her history and examination are for the most part consistent with ALS, but there has been recent rapid unexplained weight loss, serum calcium is mildly elevated, and she has chronic neck pain. Which of the following studies are indicated in evaluation of her disorder?
    a. EMG/NCS
    b. MRI of the brain and spine
    c. Anti-Hu antibody
    d. Serum parathyroid hormone
    e. All of the above
A
  1. e
    The diagnosis of amyotrophic lateral sclerosis (ALS) is made on the basis of the history, examination, and electrophysiologic testing. The El-Escorial diagnostic criteria for definite ALS as defined by the World Federation of Neurology include clinical and/or electrophysiologic evidence of both upper and lower motor neuron involvement in at least three of the four following regions: bulbar, cervical, thoracic, and lumbosacral.
    Electrodiagnostic features of ALS include evidence of ongoing denervation indicated by the presence of spontaneous activity (fibrillation potentials) and chronic denervation (reduced recruitment and polyphasic motor units). Other variable features include fasciculation potentials and motor unit instability (variability of motor unit amplitude and presence of repetitive discharges). Reduced motor unit recruitment with a rapid firing rate on EMG reflects compensation of existing motor units for those that are no longer functional, and only a single motor unit may be seen to be activated during voluntary contraction in a severely denervated muscle. Sensory NCS are normal. EMG/NCS is indicated in the evaluation of a patient with suspected ALS, and should include evaluation of cervical, thoracic, and lumbosacral regions routinely, and of bulbar region when clinically affected.
    The differential diagnosis of ALS is broad, and investigations into alternative, especially treatable, causes are indicated when there are features suggesting the possibility of another diagnosis. In the evaluation of an ALS patient, laboratory and imaging studies are done as indicated to exclude mimickers of ALS. There are a variety of other disorders that lead to clinical signs and symptoms of both upper and lower motor neuron involvement that are on the differential diagnosis of ALS, including Machado-Joseph disease (spinocerebellar ataxia type 3), adrenoleukodystrophy, multifocal motor neuropathy, hyperparathyroidism, cervical spondylosis, GM2 gangliosidoses, polyglucosan body disease, paraneoplastic motor neuron disease, HIV-associated motor neuron disease, West Nile virus infection, and others. Laboratory tests may include evaluation of parathyroid hormone, paraneoplastic autoantibodies (such as anti-Hu), GM1 autoantibodies, and others depending on the suspicion. MRI of the brain and spinal cord are indicated to exclude other potential causes masquerading clinically as ALS, such as cervical spondylosis. MRI in ALS may show increased signal in the corticospinal tract on T2-, proton density-, and FLAIR-weighted images, possibly due to Wallerian degeneration.
    Bradley WG, Daroff RB, Fenichel GM, et al. Neurology in Clinical Practice, 5th ed. Philadelphia, PA: Elsevier; 2008.
  2. a
    Copper is a trace element that functions as a prosthetic group in metalloenzymes that have a role in maintaining the structure and function of the CNS. Copper deficiency causes a myelopathy similar to that seen in vitamin B12 deficiency. It also produces multiple other neurologic manifestations, including peripheral neuropathy, CNS demyelination, myopathy, and optic neuropathy. Hematologic manifestations are also seen in copper deficiency, with anemia, neutropenia, and left shift in granulocytic and erythroid maturation. MRI of the spine may show T2-signal hyperintensity in the paramedian cord, most frequently in the cervical region.
    Copper deficiency results from malabsorption, one of the most common causes being previous gastric surgery, especially bariatric surgery for weight loss. It is frequent to find vitamin B12 deficiency along with copper deficiency, and in most cases in clinical practice, vitamin B12 is supplemented, whereas copper is usually overlooked. Lack of improvement with vitamin B12 supplementation should prompt evaluation for copper deficiency.
    Excessive zinc ingestion may also contribute to copper deficiency, because zinc induces the synthesis of metallothionein in enterocytes, which has a high affinity to copper, leading to internalization of copper into the enterocytes and its eventual loss when they are sloughed off from the mucosa.
    Kumar N. Copper deficiency myelopathy (Human Swayback). Mayo Clin Proc. 2006; 81(10):1371–1384.
    Ropper AH, Samuels MA. Adams and Victor’s Principles of Neurology, 9th ed. New York: McGraw-Hill; 2009.
31
Q
  1. A 45-year-old morbidly obese woman undergoes gastric bypass surgery. She comes to the clinic 3 years later after having lost about 40 kg, with the complaint of difficulty walking. On examination she has manifestations consistent with a myelopathy, with a spastic gait and sensory ataxia. Laboratory tests show anemia with neutropenia and a normal vitamin B12 level, but a high methylmalonic acid. After treatment with vitamin B12 she does not improve. You suspect copper deficiency. Which of the following is incorrect regarding copper deficiency?
    a. Zinc deficiency is a risk factor
    b. Copper deficiency may coexist with vitamin B12 deficiency
    c. Optic nerve involvement may be caused by copper deficiency
    d. Findings of peripheral neuropathy may be present in patients with copper deficiency
    e. The spinal cord segment most commonly affected is in the cervical region
A
  1. a
    Copper is a trace element that functions as a prosthetic group in metalloenzymes that have a role in maintaining the structure and function of the CNS. Copper deficiency causes a myelopathy similar to that seen in vitamin B12 deficiency. It also produces multiple other neurologic manifestations, including peripheral neuropathy, CNS demyelination, myopathy, and optic neuropathy. Hematologic manifestations are also seen in copper deficiency, with anemia, neutropenia, and left shift in granulocytic and erythroid maturation. MRI of the spine may show T2-signal hyperintensity in the paramedian cord, most frequently in the cervical region.
    Copper deficiency results from malabsorption, one of the most common causes being previous gastric surgery, especially bariatric surgery for weight loss. It is frequent to find vitamin B12 deficiency along with copper deficiency, and in most cases in clinical practice, vitamin B12 is supplemented, whereas copper is usually overlooked. Lack of improvement with vitamin B12 supplementation should prompt evaluation for copper deficiency.
    Excessive zinc ingestion may also contribute to copper deficiency, because zinc induces the synthesis of metallothionein in enterocytes, which has a high affinity to copper, leading to internalization of copper into the enterocytes and its eventual loss when they are sloughed off from the mucosa.
    Kumar N. Copper deficiency myelopathy (Human Swayback). Mayo Clin Proc. 2006; 81(10):1371–1384.
    Ropper AH, Samuels MA. Adams and Victor’s Principles of Neurology, 9th ed. New York: McGraw-Hill; 2009.
32
Q
  1. A 40-year-old man presents with complaints of inability to lift his arms above his head and difficulty climbing stairs. Examination shows Medical Research Council 4/5 weakness in shoulder and hip girdle muscles. He is also noted to have prominent fasciculations in his chin. A subtle high frequency postural tremor is present. His breast tissue is noted to be enlarged on examination. A family history of similar symptoms is present in his brother and two maternal cousins. His mother is apparently unaffected. Which of the following statements is correct regarding this disorder?
    a. It is autosomal recessive
    b. Precocious puberty commonly occurs
    c. Facial fasciculations are rare, but when present, suggest the diagnosis
    d. It typically presents in early childhood
    e. It results from a trinucleotide repeat expansion in the androgen receptor gene
A
  1. e
    This patient’s history and examination are consistent with Kennedy’s disease (X-linked spinobulbar muscular atrophy). Kennedy’s disease typically presents in males beginning the fourth decade of life. The clinical presentation includes motor weakness often starting in proximal muscles associated with features of lower motor neuron dysfunction, such as atrophy and hyporeflexia. Tremor, muscle cramps, and fasciculations, particularly involving the face and perioral region, are other features. Later in the course of the disease, evidence of bulbar dysfunction becomes apparent. Examination may reveal gynecomastia, but its absence does not preclude this diagnosis. Endocrine abnormalities, including hypogonadism with sterility and diabetes, occur in some patients. This disorder rarely manifests in females, likely due to random X-inactivation, but has been reported, presenting with bulbar dysfunction. Kennedy’s disease results from expansion of a CAG repeat in the androgen receptor protein gene on the X chromosome. Genetic testing for this disorder is commercially available. Treatment is supportive.
    Bradley WG, Daroff RB, Fenichel GM, et al. Neurology in Clinical Practice, 5th ed. Philadelphia, PA: Elsevier; 2008.
33
Q
  1. Regarding the spinal cord, which of the following is incorrect?
    a. It has 31 pairs of spinal nerves: 8 cervical, 12 thoracic, 5 lumbar, 5 sacral, and 1 coccygeal
    b. It lies within the spinal canal ventral to the posterior longitudinal ligament
    c. In adults, it ends at the level of L1 to L2
    d. There are two enlargements, one at the cervical level and another at the lumbosacral level
    e. The denticulate ligament extends from pia mater to dura mater on both sides of the spinal cord
A
  1. b
    The spinal cord lies dorsal to the posterior longitudinal ligament.
    The vertebral column is composed of vertebral bodies, pedicles, and laminae. Two pedicles arise from the posterior aspect of each vertebral body. Two laminae fuse posteriorly leaving the spinal canal in the middle within which the spinal cord lies. Multiple ligaments attach and give support to the vertebral column. The anterior longitudinal ligament runs along the anterior surface of the vertebral bodies, whereas the posterior longitudinal ligament runs along the posterior surface and ventral to the spinal cord. The ligamentum flavum runs in the posterior aspect of the spinal canal, and the denticulate ligament extends on both sides of the spinal cord between the pia mater and the dura mater. Between each vertebral body there is an intervertebral disc, which is composed of a central nucleus pulposus and a surrounding annulus fibrosus.
    The spinal cord has 31 segments and a corresponding number of pairs of spinal nerve roots, with 8 cervical, 12 thoracic, 5 lumbar, 5 sacral, and 1 coccygeal. The cervical nerve roots exit the spinal column above the vertebral body of the same number, except for C8, which exit between the C7 and T1 intervertebral foramen. The subsequent nerve roots below this level exit the spinal column below the corresponding vertebral body.
    In adults, the spinal cord extends from the foramen magnum to the level of L1 to L2. It has two major enlargements, one in the cervical region and another one in the lumbosacral region. These enlargements correlate with the areas related to innervation of the arms and legs.
    Blumenfeld H. Neuronatomy through Clinical Cases, 1st ed. Sunderland, MA: Sinauer Associates; 2002.
    Ropper AH, Samuels MA. Adams and Victor’s Principles of Neurology, 9th ed. New York: McGraw-Hill; 2009.
34
Q
  1. At birth, a male baby is noted to be hypotonic, admitted to the ICU and intubated for respiratory failure, and he eventually undergoes tracheostomy. He is noted to have little movement of the extremities and persistent head lag. Muscle biopsy is obtained and the findings are shown in Figure 11.5. What is the most likely diagnosis in this patient?

a. Spinal muscular atrophy (SMA) type 1
b. SMA type 4
c. Charcot-Marie-Tooth type 1a
d. Congenital myasthenia
e. Neonatal botulism

A
  1. a
    This patient’s history and muscle biopsy are consistent with spinal muscular atrophy (SMA) type 1. The SMAs are a group of four disorders of anterior horn cell degeneration. SMA type 1, infantile SMA, or Werdnig-Hoffman disease presents with decreased fetal movements, neonatal hypotonia, and weak cry. Patients with SMA type 1 exhibit head lag and frog-leg posture when supine, and are never able to sit up or achieve anti-gravity strength in the arms or legs. Diffuse areflexia is often present. Bulbar involvement leads to dysphagia, and respiratory involvement occurs. Death usually occurs by 2 years of age.
    In SMA type 2, intermediate SMA, symptoms begin in the first 1 to 2 years of life, with motor delay and tremor in some cases. This form is less severe than SMA type 1, with most children being able to sit unsupported, but significant contractures develop, and most are not able to ambulate.
    SMA type 3, juvenile SMA or Kugelberg-Welander disease, typically presents in childhood (between ages 5 and 15 years) with difficulty walking. The clinical picture is one of proximal muscle weakness, a fine action tremor, and fasciculations. Patients often remain ambulatory into adulthood.
    SMA type 4, adult-onset SMA or pseudomyopathic SMA, is the least common and least severe. Symptoms typically begin in the third to fourth decades of life and include proximal muscle weakness, with the quadriceps being prominently involved, and fasciculations. Ambulation into late adulthood is common. Cranial nerve and respiratory involvement is rare.
    Commercially available molecular analysis of the survival motor neuron (SMN) gene is available. Serum creatine kinase may be significantly elevated (more than 10 times normal), particularly in the younger-onset forms. Sensory NCS are normal; EMG shows evidence of acute and chronic denervation and reinnervation, with large polyphasic motor units. Complex repetitive discharges occur in SMA type 3. Muscle biopsy shows atrophy of the entire fascicles or groups of fascicles, with normal or hypertrophied neighboring fascicles, as depicted in Figure 11.5.
    The majority of cases of SMA are due to mutations in the SMN1 gene on chromosome 5. Inheritance is autosomal recessive in most cases, though a minority of cases involving other genes with autosomal dominant or X-linked inheritance have been identified. The gene product, SMN1 protein, is involved in RNA processing. In later-onset forms, some residual functional protein is present, leading to the milder phenotype.
    As mentioned, SMA type 4 presents in adulthood. Charcot-Marie-Tooth type 1 typically presents in childhood (rather than the neonatal period), and will not be associated with the muscle biopsy findings shown in Figure 11.5. Congenital myasthenia and botulism are on the differential diagnosis of neonatal hypotonia but would not lead to the muscle biopsy findings shown in Figure 11.5.
    Aminoff MJ. Neurology and General Medicine, 4th ed. Philadelphia, PA: Elsevier; 2008.
    Bradley WG, Daroff RB, Fenichel GM, et al. Neurology in Clinical Practice, 5th ed. Philadelphia, PA: Elsevier; 200
35
Q
  1. A 60-year-old anesthetist with obesity, hypertension, and diabetes undergoes gastric bypass surgery, and an inhaled anesthetic is used during the procedure. Around 5 days after the surgery he notices that he cannot move his legs, and he has a strange sensation below his waist. An MRI of the thoracic spine shows T2 cord signal hyperintensity with no evidence of an extradural collection. Which of the following is the most likely diagnosis?
    a. Copper deficiency
    b. Folate deficiency
    c. Spinal epidural hematoma
    d. Spinal epidural abscess
    e. Nitrous oxide toxicity
A
  1. e
    This patient has nitrous oxide toxicity, causing a neurologic manifestation that has been termed “anesthesia paresthetica.” Nitrous oxide is an inhalational anesthetic agent that produces irreversible oxidation of cobalamin and makes the methylcobalamin inactive. The manifestations occur in patients with underlying cobalamin deficiency even after a single exposure to nitrous oxide, usually with a rapid onset of symptoms. However, chronic exposure can lead to similar manifestations, and this has been described among anesthetists, dentists, or medical personnel working with nitrous oxide, either from occupational exposure or from abuse. Patients with chronic exposure can have an acute exacerbation during an acute exposure to larger amounts of nitrous oxide.
    The neurologic manifestations include myelopathy, neuropathy, or even cognitive changes and encephalopathy. MRI changes with T2 hyperintensity can be detected. Prophylactic vitamin B12 can help prevent this condition.
    This patient does not have copper deficiency, which usually produces a more protracted course, usually related to malabsorption. In cases of malabsorption after gastric bypass, the manifestations usually occur gradually over months or years. Folate deficiency causes neurologic manifestations in very rare instances, and usually gradually over long periods of time in patients who have other nutrient deficiencies.
    In cases of spinal epidural abscess or hematomas causing myelopathy, the MRI images usually are distinct, showing spinal cord compression, which is not the case in this patient.
    Kinsella LJ, Green R. “Anesthesia paresthetica”: Nitrous oxide-induced cobalamin deficiency. Neurology. 1995; 45:1608–1610.
    Kumar N. Copper deficiency myelopathy (Human Swayback). Mayo Clin Proc. 2006; 81:1371–1384.
    Ropper AH, Samuels MA. Adams and Victor’s Principles of Neurology, 9th ed. New York: McGraw-Hill; 2009.
36
Q
  1. An 18-year-old man, originally from Hawaii, presents with a history of 2 years of progressive bilateral upper extremity weakness, right worse than left. He has no pain, and denies dysphagia or dysarthria. On examination he has weakness, atrophy, and fasciculations, more prominent in the hand muscles, right worse than left. His lower extremities are not affected. His sensory examination is normal. He also has a past medical history of seasonal allergies. On follow-up 3 years later, his hand weakness is stable to minimally improved, and no weakness has developed in other areas. What is the most likely diagnosis in this patient?
    a. Amyotrophic lateral sclerosis
    b. Primary lateral sclerosis
    c. Hirayama disease
    d. Spinal muscular atrophy
    e. Cervical osteoarthritis
A
  1. c
    This patient has Hirayama disease, also known as monomelic amyotrophy. This condition presents in young patients of Asian origin, and it is characterized by progressive asymmetric wasting of one or both hands and forearms. Examination shows atrophy and fasciculations with no sensory deficits. These patients have a high incidence of atopic disorders. The pathophysiology is not understood, but some have suggested a chronic progressive compression of the cervical cord, associated with dynamic changes of the cervical dural sac and spinal cord induced by neck flexion. MRI of the cervical spine in flexion demonstrates cervical cord thinning with signal changes. A cervical collar may be used to prevent neck flexion, and surgical intervention may have an impact in some patients.
    Amyotrophic lateral sclerosis (ALS) is a progressive degenerative motor neuron disease in which there are upper and lower motor neuron signs in multiple segments of the neuraxis. This patient has only lower motor neuron findings in his upper extremities, which is usually not consistent with ALS.
    Primary lateral sclerosis is a disorder in which the predominant feature is the presence of upper motor neuron signs, without lower motor neuron signs, which is not consistent with the case depicted.
    Spinal muscular atrophy is a group of genetic disorders that affect the anterior horn cells of the spinal cord and motor nuclei of the brain stem. Usually there are findings affecting upper and lower limbs, as well as bulbar muscles.
    This patient is not in the age group and does not have other clinical features to suggest the diagnosis of cervical osteoarthritis.
    Fenichel GM. Clinical Pediatric Neurology: A Signs and Symptoms Approach, 6th ed. Philadelphia, PA: Saunders Elsevier; 2009.
    Hirayama K. Juvenile muscular atrophy of distal upper extremity (Hirayama disease). Inter Med. 2000; 39:283–290.
    Ropper AH, Samuels MA. Adams and Victor’s Principles of Neurology, 9th ed. New York: McGraw-Hill; 2009.
37
Q
  1. A 77-year-old man with a history of hypertension, diabetes, hyperlipidemia, and coronary disease is admitted for open repair of a thoracoabdominal aortic aneurysm. When he wakes up from surgery, he is in pain, cannot move his legs, and does not have sensation to pinprick over his legs; however, he is able to feel vibration and proprioception in his toes. Which of the following is correct regarding this case?
    a. The patient has nitrous oxide toxicity–related myelopathy
    b. A spinal epidural hematoma is the most likely cause
    c. Vitamin B12 deficiency is the most likely cause
    d. Copper deficiency is the most likely cause
    e. This patient has a spinal cord infarct
A
  1. e
    This patient has a spinal cord infarction in the territory of the anterior spinal artery. The spinal cord’s blood supply consists of one anterior spinal artery and two posterior spinal arteries. The former supplies the anterior two-thirds of the spinal cord, whereas the two posterior spinal arteries supply the posterior one-third. Therefore patients with an anterior spinal artery syndrome present with weakness below the lesion, with sensory deficit to pain and temperature due to involvement of the lateral spinothalamic tracts, but because the dorsal columns are not involved, there is preservation of vibratory sense and proprioception.
    Spinal cord infarction may occur after aortic aneurysm surgical repair. Endovascular approaches have a lower rate of overall complications as compared to open surgical approaches, including lower rates of spinal cord ischemia.
    In this case there is no history or clinical findings to support a spinal epidural hematoma as the cause. Patients with copper and vitamin B12 deficiency usually present rather gradually and not rapidly as in this case. Furthermore, vitamin B12 deficiency is associated with posterior column degeneration affecting proprioception and vibratory sensation, which was not present in this case. Patients with vitamin B12 deficiency are at risk for nitrous oxide toxicity.
    Ropper AH, Samuels MA. Adams and Victor’s Principles of Neurology, 9th ed. New York: McGraw-Hill; 2009.
38
Q
  1. A 22-year-old man, intravenous drug user, reports a history of 1 month of low-grade fevers and back pain, and over the past 3 days, he has developed progressive lower extremity weakness and urinary incontinence. His temperature is 39°C, and on examination he has paraplegia and a sensory level at T8. Rectal tone is decreased. Which of the following is correct regarding this patient’s most likely condition?
    a. A lumbar puncture is indicated, and cultures are positive in the majority of the cases
    b. MRI of the spine is not very sensitive for the diagnosis of this condition
    c. Neurosurgical consultation may not be required if fever resolves with initial antibiotic therapy
    d. Staphylococcus aureus is the most common pathogen
    e. The treatment includes broad-spectrum antibiotic therapy for 2 weeks
A
  1. d
    This patient most likely has a spinal epidural abscess. This condition occurs when an infectious process extends into the epidural space, either from local extension or from a remote source via hematogenous spread. Risk factors include diabetes mellitus, intravenous drug use, previous infection, history of spine injury or spine surgery, renal disease, and multiple medical comorbidities. The most common presenting symptoms are fever, low back pain, and progressive neurologic symptoms attributed to cord or root compression. Myelopathic findings develop over time, and patients may have upper motor neuron findings below the level of the lesion. Sometimes the patient worsens rapidly and examination may show findings of spinal shock. The symptoms may ascend over time as the epidural abscess propagates cranially.
    MRI of the spine is the standard test for diagnosis and has high sensitivity. An LP is not usually performed, and may be contraindicated due to risk of spread of infection to the intrathecal compartment. Furthermore, CSF cultures are positive only in 25% of cases.
    The most common organism is S. aureus, followed by streptococci, gram-negative bacilli, and anaerobic organisms. Empiric broad-spectrum antibiotics should be started early, with narrowing of the spectrum once a specific organism is identified. The usual duration of antibiotic therapy is between 4 and 6 weeks; however, antibiotics alone may not be effective. Once this condition is suspected, urgent neurosurgical consultation should be obtained. If the patient has findings of cord compression, surgical evacuation should be performed. Epidural spinal abscess is a true neurosurgical and neurologic emergency.
    Ropper AH, Samuels MA. Adams and Victor’s Principles of Neurology, 9th ed. New York: McGraw-Hill; 2009.
39
Q

Review

A
40
Q
  1. A 52-year-old man presents with progressive lower extremity weakness over the past 6 months. He reports that last week he woke up one day and his legs could not move at all. His MRI is shown in Figure 2.19. A vascular anomaly is suspected. Which of the following is the most likely diagnosis?

FIGURE 2.19 Sagittal STIR MRI

a. Epidural hematoma
b. Cavernous malformation
c. DAVF
d. Venous angioma
e. Spinal cord infarct

A
  1. c
    This patient has a spinal dural arteriovenous fistula (DAVF), which is the most common type of spinal vascular malformation. This lesion can be fed by one or multiple arteries and is a low pressure and low flow vascular lesion. DAVF is more frequent in men and above 50 years of age. The typical location is in the lower thoracic and lumbar regions, and this type of lesion rarely produces hemorrhage. Patients present with pain, weakness, and sensory symptoms below the level of the lesion. The myelopathic syndrome is usually gradually progressive and caused by venous hypertension and congestion. MRI of the spine shows enlargement of the cord with hyperintensity on T2 seen over several levels, with intradural flow voids suggesting this pathology (Figure 2.19). Spinal angiogram is the gold standard diagnostic test to detect this abnormality. Treatment involves angiographic embolization and/or surgical removal of the lesion. The radiographic findings in this case are consistent with an expanding lesion with T2 hyperintensity in the cord. The presentation is gradually progressive and consistent with DAVF.
    Epidural hematoma and spinal cord infarct have a more acute presentation and do not show the MRI findings seen in this case. This is not a cavernous malformation or a venous angioma, which are not common in this location.
    Bradley WG, Daroff RB, Fenichel GM, Jankovic J. Neurology in Clinical Practice, 5th ed. Philadelphia, PA: Elsevier; 2008.
41
Q
  1. Which opiate receptor class is involved with spinal analgesia?
    a. Kappa (κ)
    b. Beta (β)
    c. Mu (μ)
    d. Delta (δ)
    e. Nociceptin (ORL1)
A

6a. Opioid receptors are a group of G-protein coupled receptors, with opioids acting as ligands. The endogenous opioids are dynorphins, enkephalins, endorphins, endomorphins, and ORL1. There are four major subtypes of opioid receptors. The first is δ, including subtypes δ1 and δ2. They are involved with analgesia, antidepressant effects, and physical dependence. The second is κ and includes κ1, κ2, and κ3. These are involved in spinal analgesia, sedation, miosis, and inhibition of antidiuretic hormone release. The third is μ and includes μ1, μ2, and μ3. The subtype μ1 is involved in supraspinal analgesia and physical dependence; μ2 is involved in respiratory depression, miosis, euphoria, reduced gastrointestinal motility, and physical dependence. The actions of μ3 are not clear. The fourth is ORL1/orphanin, which is involved in anxiety, depression, appetite, and development of tolerance to μ agonists.

42
Q
  1. A 29-year-old woman presents with a slowly progressive spastic paraparesis of her legs over 2 years. In addition to this finding on examination, you also see that she has absent vibratory sense and proprioception in her feet. She also mentions that she takes zinc supplementation twice daily for the last several years to prevent “colds.” Which of the following is the most likely etiology of these findings?
    a. Vitamin E deficiency
    b. Vitamin B12 deficiency
    c. Copper excess
    d. Zinc deficiency
    e. Copper deficiency
  2. Which of the following would be one of your primary treatment recommendations for the patient depicted in question 22?
    a. Vitamin B12 replacement therapy
    b. Chelation therapy
    c. Copper supplementation
    d. Increase zinc supplementation
    e. Vitamin E supplementation
A
  1. e, 23. c
    This patient has copper deficiency related to excess zinc intake. This can be related to excess dietary intake (as in this case), overuse of denture cream, parenteral feeding deficiency, and gastrointestinal surgery. This syndrome occurs because zinc increases enterocyte metallothionein synthesis. These excess enterocyte metallothioneins easily bind copper. The resultant excessively bound copper within the enterocytes is then excreted when the enterocytes are sloughed off, resulting in impaired absorption. Copper deficiency myelopathy syndrome resembles the subacute combined degeneration seen with vitamin B12 deficiency. Copper deficiency causes a sensorimotor peripheral neuropathy with axonal loss features on electrodiagnostic studies combined with myelopathy in the form of spastic paraparesis and posterior column dysfunction. Pancytopenia is also frequently associated. Copper deficiency is discussed also in Chapter 11. The history of excess zinc makes this more likely than vitamin B12 deficiency in this particular patient. Vitamin E deficiency can mimic symptoms of spinocerebellar ataxia, which is not present in this patient.
    Goodman BP, Bosch EP, Ross MA, et al. Clinical and electrodiagnostic findings in copper deficiency myeloneuropathy. J Neurol Neurosurg Psychiatry. 2009; 80:524–547.
    Kumar N, Gross JB Jr, Ahlskog JE. Copper deficiency myelopathy produces a clinical picture like subacute combined degeneration. Neurology. 2004; 63:33–39.
43
Q
  1. A 38-year-old woman with Crohn’s disease, chronic diarrhea, and steatorrhea is referred to your office for evaluation of gait instability and ataxia. On examination, you find that she has some dysmetria, mild dysarthria, and absent reflexes. What is the likely cause of these neurologic findings?
    a. Vitamin A deficiency
    b. Vitamin D deficiency
    c. Vitamin B12 deficiency
    d. Copper deficiency
    e. Vitamin E deficiency
  2. Which of the following would be one of your primary treatment recommendations for the patient depicted in question 24?
    a. Vitamin E supplementation
    b. Vitamin A supplementation
    c. Copper replacement
    d. Increased sunlight and vitamin D supplementation
    e. Vitamin B12 replacement
A
  1. e, 25. a
    This patient’s neurologic findings are likely the result of vitamin E deficiency related to chronic diarrhea and subsequent malabsorption of fat-soluble vitamins, that is, vitamins D, A, K, and E. These vitamins need to be supplemented, especially vitamin E as in this case. Symptoms resemble a spinocerebellar ataxia syndrome such as Friedrich ataxia and may include ataxia, dysarthria, areflexia, extensor plantar responses, and large fiber sensory loss. Presentation can occur at any age in the setting of chronic diarrhea and malabsorption diseases. Vitamin E deficiency is more likely to occur in childhood when a genetic etiology is present, such as α-tocopherol-transfer protein mutation or abetalipoproteinemia (Bassen-Kornzweig syndrome). Abetalipoproteinemia is caused by a mutation of a microsomal triglyceride transfer protein resulting in absence of apolipoprotein B–containing proteins. Laboratory findings show low vitamin E levels. Acanthocytosis may also be seen on peripheral smear. See discussion to question 22 for copper deficiency, and that to question 27 for vitamin B12 deficiency.
    Bradley WG, Daroff RB, Fenichel GM, et al. Neurology in Clinical Practice, 5th ed. Philadelphia, PA: Elsevier; 2008.
    Ropper AH, Samuels MA. Adams and Victor’s Principles of Neurology, 9th ed. New York, NY: McGraw-Hill; 2009.
    Satya-Murti S, Howard L, Krohel G, et al. The spectrum of neurologic disorder from vitamin E deficiency. Neurology. 1986; 36:917–921.
44
Q
  1. Which of the following would be the least likely finding related to thiamine (vitamin B1) deficiency?
    a. Neuropathy
    b. Arrhythmia
    c. Extensor plantar responses
    d. Congestive heart failure
    e. Wernicke-Korsakoff syndrome
A
  1. c
    Extensor plantar responses would not be seen in thiamine (vitamin B1) deficiency. This deficiency would be more likely to cause peripheral neuropathy and flexor plantar responses. Thiamine deficiency in Wernicke-Korsakoff syndrome is discussed in question 10. Thiamine deficiency causes an axonal, sensorimotor peripheral neuropathy with weakness and distal sensory loss. This is termed “dry beriberi.” When it is associated with cardiac involvement in the form of cardiomegaly, cardiomyopathy, congestive heart failure, arrhythmia and tachycardia, and peripheral edema, it is called “wet beriberi.” Thiamine deficiency has also been reported to cause Leigh syndrome (Leigh subacute necrotizing encephalomyelopathy). Laboratory findings may include decreased serum thiamine, erythrocyte transketolase activity and urinary thiamine, with increased pyruvate and lactate levels, in addition to characteristic electrodiagnostic findings.
    Bradley WG, Daroff RB, Fenichel GM, et al. Neurology in Clinical Practice, 5th ed. Philadelphia, PA: Elsevier; 2008.
    Pincus JH. Subacute necrotizing encephalomyelopathy (Leigh’s disease): a consideration of clinical features and etiology. Dev Med Child Neurol. 1972; 14:87–101.
    Tanphaichitr V. In: Shils M (Ed), Modern Nutrition in Health and Medicine, 9th ed. Philadelphia, PA: Lippincott Williams & Wilkins; 2000.
45
Q
  1. A defect in production of which of the following underlies the pathophysiology of the neurologic findings in vitamin B12 deficiency?
    a. Threonine
    b. Serotonin
    c. Homocysteine
    d. Methionine
    e. Serin
A
  1. d
    The neurologic deficits in vitamin B12 (cobalamin) deficiency are ultimately related to a defect in methionine production. Vitamin B12 is a cofactor for methionine synthase, which is involved in conversion of homocysteine to methionine and production of succinyl-CoA from methylmalonyl-CoA. Methionine is subsequently a precursor for S-adenosyl-l-methionine, which helps with methylation of myelin basic protein. Without this process, abnormal myelin structure results in neurologic deficit. The syndrome resulting from vitamin B12 deficiency is called subacute combined degeneration because of sensorimotor peripheral neuropathy combined with myelopathy, spastic paraparesis and posterior column dysfunction. Complete blood cell count may reveal a macrocytic anemia. Occasionally, vitamin B12 levels may be in the low normal or even normal range despite true deficiency. If clinical suspicion is present for deficiency, the levels of metabolic intermediaries homocysteine and methylmalonic acid should be checked, both of which would be elevated in vitamin B12 deficiency. Animal products (meat and dairy) provide the primary dietary source of vitamin B12 for humans. This puts older adults, alcoholics, patients with malnutrition, and strict vegans at high risk for development of deficiency.
    Bradley WG, Daroff RB, Fenichel GM, et al. Neurology in Clinical Practice, 5th ed. Philadelphia, PA: Elsevier; 2008.
    Green R, Kinsella LJ. Current concepts in the diagnosis of cobalamin deficiency. Neurology. 1995; 45(8):1435–1440.
    Hemmer B, Glocker FX, Schumacher M, et al. Subacute combined degeneration: clinical, electrophysiological, and magnetic resonance imaging findings. J Neurol Neurosurg Psychiatry. 1998; 65:822–827.
    Lindenbaum J, Savage DG, Stabler SP, et al. Diagnosis of cobalamin deficiency, II: relative sensitivities of serum cobalamin, methylmalonic acid, and total homocysteine concentrations. Am J Hematol. 1990; 34:99–107.
46
Q
  1. A 49-year-old woman and her very anxious husband present to your office with progressively worsening sensory loss and weakness, which began distally in her legs and moved proximally beginning approximately a week ago. You notice she has a diffuse macular rash that is pruritic and scaling on her palms and soles. Records indicate that she had presented to the emergency department 3 weeks prior with complaints of severe nausea, vomiting, and diarrhea, and the physician had noted that her breath smelled strongly of garlic. What do you suspect is the cause for these symptoms?
    a. Guillain-Barré syndrome
    b. Cyanide poisoning
    c. Thallium poisoning
    d. Arsenic poisoning
    e. Mercury poisoning
  2. For the patient depicted in question 28, what test finding would have confirmed your suspicion if it had been completed during her emergency department visit 3 weeks prior?
    a. LP revealing albumino-cytologic dissociation
    b. Elevated urinary arsenic level
    c. Elevated urinary thallium level
    d. Elevated urinary mercury level
    e. Elevated urinary cyanide level
A
  1. d, 29. b
    This patient exhibits symptoms consistent with arsenic poisoning. Arsenic is a naturally occurring element most commonly incorporated into organic or inorganic compounds, both of which are very toxic. It can also occur in gas form. With acute exposure, symptoms may develop within minutes to hours and usually begin with gastrointestinal symptoms such as abdominal pain, nausea, vomiting, and diarrhea. A garlic odor on the breath is characteristic. These symptoms can be followed by hypotension, dehydration, and cardiac and respiratory instability. Delirium, encephalopathy, coma, and seizures may occur. Other acute manifestations include proteinuria, hematuria, and acute tubular necrosis. If patients survive, within 1 to 3 weeks, they can develop hepatitis, pancytopenia, and a symmetric sensorimotor peripheral neuropathy, which typically begins with distal paresthesias, followed rapidly by an ascending sensory loss and weakness, which mimics Guillain-Barré syndrome. The neuropathy can progress to intense burning pain, especially in the soles. In addition, dermatologic lesions can occur and may include alopecia, oral mucosal ulcerations, diffuse pruritic macular rash, and scaly rash on the palms and soles. A dry hacking cough and Mees lines (horizontal 1- to 2-mm white lines on the nails) may also occur. In chronic poisoning, the peripheral neuropathy and dermatologic symptoms are usually more prominent than the gastrointestinal symptoms. Cancers of the liver, bladder, kidney, skin, lung, nasal mucosa, and prostate have been reported with chronic exposure.
    After a suspected acute ingestion of arsenic, abdominal radiographs may reveal gastrointestinal radiopaque material. Urine arsenic levels are preferable to blood arsenic levels, but both can be used. Fish or shellfish intake within the previous 48–72 hours can cause falsely elevated levels of arsenic. For chronic exposure, hair and nail samples can be analyzed for the presence of arsenic, and 24-hour urine arsenic or spot urine arsenic and creatinine levels can be checked. Additional evaluations should include renal and liver function tests, complete blood cell count, urinalysis, and electrodiagnostic testing if there are symptoms of peripheral neuropathy. A distal sensorimotor axonopathy is the typical finding.
    Acute treatment includes fluid and electrolyte replacement, cardiac monitoring, activated charcoal, and chelation therapy. Chelation agents typically used include dimercaprol (British Anti-Lewisite) and meso-2,3-dimercaptosuccinic acid (succimer).
    Although symptoms can mimic Guillain-Barré syndrome, the constellation of clinical symptoms and signs described and adequate evaluation should have ruled this out. Cyanide and mercury poisoning are discussed in questions 30 and 31 respectively. Thallium causes acute gastrointestinal symptoms, confusion, painful (mostly sensory) neuropathy with autonomic features, and alopecia, which classically occurs about 2 to 4 weeks after ingestion.
    Danan M, Dally S, Conso F. Arsenic-induced encephalopathy. Neurology. 1984; 34:1524.
    Flomenbaum N, Goldfrank L, Hoffman R, et al. Goldfrank’s Toxicologic Emergencies, 8th ed. New York, NY: McGraw-Hill; 2006.
    Windebank AJ. Arsenic. In: Spencer PS, Schaumburg HH (Eds), Experimental and Clinical Neurotoxicology. New York, NY: Oxford University Press; 2000.
    Yip L, Dart RC. Arsenic. In: Sullivan JB, Krieger GR (Eds), Clinical Environmental Health and Toxic Exposures. Philadelphia, PA: Lippincott Williams & Wilkins; 2001.
47
Q
  1. A 67-year-old man who lives on a farm with his wife presents with complaints of blurred and double vision, difficulty swallowing, and neck and shoulder weakness. There is no diurnal variation. These symptoms have been progressively worsening over the last 2 days. He reports being very healthy otherwise and attributes his good health to growing his own food and storing all excess by canning. On examination, you find multiple cranial nerve abnormalities and fixed pupillary dilation, and he is unable to contract his deltoids against resistance. What diagnosis would you first suspect?
    a. Myasthenia gravis
    b. Lambert-Eaton myasthenic syndrome
    c. Botulism
    d. Guillain-Barré syndrome
    e. Leptomeningeal carcinomatosis
A
  1. c
    This patient describes symptoms most consistent with botulism, likely related to foodborne source from home canning. Botulism is a potentially life-threatening neuroparalytic syndrome resulting from exposure to botulinum toxin, produced by Clostridium botulinum. There are at least eight distinct types of botulinum toxin, although the most commonly involved is botulinum toxin A. There are multiple forms of botulism including foodborne botulism (ingestion of food contaminated by botulinum toxin), wound botulism (infection of a wound by C. botulinum, with subsequent production of neurotoxin), infantile botulism (ingestion of clostridial spores that then colonize the gastrointestinal tract and release toxin), adult enteric infectious botulism (toxin produced in the gastrointestinal tract), and inhalational botulism (aerosolized toxin related to acts of bioterrorism).
    Botulinum toxin binds to the synaptotagmin II receptor on presynaptic cholinergic synapses and neuromuscular junctions. After the heavy chain of the toxin binds to the receptors, the light chain translocates into the nerve cell via endocytosis. Upon entering the cytoplasm, the toxin irreversibly inhibits acetylcholine release by cleaving various proteins involved in neuroexocytosis of acetylcholine. Botulinum toxin A and E cleave SNAP-25; botulinum toxin C cleaves SNAP-25 and syntaxin; and botulinum B, D, F, and G cleave synaptobrevin. Reversal of this inhibition requires sprouting of a new presynaptic terminal and formation of a new synapse. This generally takes 3 to 6 months. Although the toxin can be quite harmful, these effects are used for therapeutic purposes, such as for the treatment of dystonia, spasticity, and other neurologic disorders.
    Symptoms related to foodborne botulism may begin within 12 to 36 hours after ingestion of the toxin but may be delayed for several days. Symptoms include acute onset of multiple cranial neuropathies, blurred vision (due to fixed pupillary dilation), symmetric descending weakness, urinary retention, and constipation. Gastrointestinal symptoms such as diarrhea, abdominal pain, nausea, and vomiting often precede neurologic symptoms in foodborne botulism.
    Symptoms are not consistent with myasthenia gravis (there is no diurnal variation or other historical points to suggest fatigability; discussed in Chapter 10), Lambert-Eaton myasthenic syndrome (discussed in Chapter 10), or Guillain-Barré syndrome (which presents with ascending, not descending, weakness and without pupillary involvement; discussed in Chapter 9). He has no history of cancer and is otherwise healthy, making leptomeningeal carcinomatosis unlikely, especially with the abrupt onset of symptoms.
    Abrutyn E. Botulism. In: Fauci AS, Isselbacher KJ, Braunwald E, et al. (Eds), Principles of Internal Medicine, 14th ed. New York, NY: McGraw-Hill; 1998.
    Hughes JM, Blumenthal JR, Merson MH, et al. Clinical features of types A and B food-borne botulism. Ann Intern Med. 1981; 95:442–445.
    Jin R, Rummel A, Binz T, et al. Botulinum neurotoxin B recognizes its protein receptor with high affinity and specificity. Nature. 2006; 444(7122):1019–1020.
48
Q
  1. Regarding the neurotransmitter GABA, which of the following is incorrect?
    a. GABA is the main excitatory neurotransmitter in the central and peripheral nervous system, along with glycine
    b. The GABAA receptor is an ionotropic receptor and the GABAB receptor is a metabotropic receptor
    c. The medication baclofen is a selective agonist at GABAB receptors
    d. Benzodiazepines act at the GABAA receptor
    e. GABA is an example of an amino acid that acts as a neurotransmitter
A
  1. a
    GABA is an amino acid and the major inhibitory neurotransmitter in the CNS. Glycine is also an inhibitory amino acid neurotransmitter and plays a prominent role in the brain and spinal cord. GABA is synthesized from glutamic acid by action of the enzyme glutamic acid decarboxylase. Disorders of this enzyme lead to a deficiency in GABA and subsequently over-activation in the CNS, as is seen in stiff-person syndrome. The GABAA receptor is an example of an ionotropic receptor, activation of which leads to opening of chloride channels. The GABAB receptor is an example of a metabotropic receptor, which is coupled to an inhibitory G protein, inhibiting adenylyl cyclase. Baclofen is an example of a selective GABAB receptor agonist; benzodiazepines act at GABAA receptors.
    Brunton LL, Lazo JS, Parker KL (Eds). Goodman and Gilmans’ Pharmacological Basis of Therapeutics, 11th ed. New York: McGraw-Hill; 2005.
49
Q
  1. A 24-year-old right-handed Caucasian woman comes to your clinic for the first time. One year ago she had one episode of a tingling sensation ascending to the mid-chest region lasting 2 weeks. One month ago she lost vision in her right eye and had pain behind the eye, worse with movement, that resolved incompletely after 2 weeks. Her MRI of the brain and cervical spine are shown in Figures 7.1 and 7.2. This presentation and MRI are consistent with:

FIGURE 7.1 Axial FLAIR MRI

FIGURE 7.2 Sagittal T2-weighted MRI

a. A clinically isolated syndrome of demyelination
b. Primary progressive multiple sclerosis
c. Acute disseminated encephalomyelitis
d. Fulminant multiple sclerosis because of two episodes in a year
e. Relapsing-remitting multiple sclerosis
2. Other tests that may be useful diagnostically for this problem include:
a. Laboratory testing to exclude a vitamin B12 deficiency and lupus erythematosus
b. Skin biopsy for small-fiber neuropathy
c. Brainstem auditory-evoked potentials
d. CSF analysis for cells, protein, glucose, and oligoclonal bands
e. a and d
3. What is correct regarding the prognosis?
a. Will progress to using a wheelchair within 5 years
b. Has benign multiple sclerosis and will not progress
c. Will likely be unable to walk after 15 years
d. Should avoid pregnancy due to worsening of her disease
e. Will likely have measurable disability on neurologic examination after 10 years

A
  1. e, 2. e, 3. e
    This patient has relapsing-remitting multiple sclerosis (RRMS). Most patients with this disorder have onset between age 20 and 40, though age of onset ranges from early childhood to late adulthood. Females are affected more than males in a 3:2 ratio. It is more common further north and south of the equator, and more common in Caucasians. RRMS is defined by relapses consistent with demyelination, with or without clinical improvement, but with episodes of clinical stability in between relapses. The term “remitting” is confusing as these patients may not return to clinical normalcy after a relapse. In addition, on MRI, they may have new lesion formation while remaining clinically stable five to ten times as often as they have new clinical episodes. A “relapse” is defined as an episode of new or worsened neurologic symptoms, lasting more than 24 hours, not due to fever or infection. Other synonyms for this are an attack, bout, or exacerbation. The diagnosis is based on two relapses separated in time with clinical evidence of multiple lesions in the central nervous system. The diagnosis can also be made on the basis of multiple lesions on the MRI in combination with an appropriate history and examination, new lesion formation on MRI over time, and new clinical activity over time.
    A clinically isolated syndrome of demyelination is a single clinical episode consistent with demyelination, but no “second episode” to make a clinical diagnosis of multiple sclerosis. Primary progressive multiple sclerosis usually occurs in older patients. These patients have a gradually progressive course from onset and do not have relapses. They may have a lower burden of MRI lesions than do patients with RRMS. Acute demyelinating encephalomyelitis is an acute disorder, more common in childhood, and often occurring soon after an infection or vaccination. Patients develop a subacute severe disorder with multifocal demyelination in the brain and spinal cord (see discussion to questions 19 to 21). Fulminant multiple sclerosis indicates a rapidly progressive course with repeated relapses that are refractory to standard treatment.
    There are no laboratory tests that are presently useful in the diagnosis of multiple sclerosis. On occasion, other disorders can mimic multiple sclerosis. Diseases commonly tested for in early evaluation of multiple sclerosis include vitamin B12 deficiency, lupus, Lyme disease in endemic areas, and other inflammatory disorders depending on clinical suspicion. Skin biopsy for small-fiber neuropathy may be useful for patients with paresthesias that affect the limbs and face, that are persistent, and in whom there is no evidence to suggest central demyelination. Brainstem auditory-evoked potentials can be abnormal in the population with multiple sclerosis, but generally are not helpful diagnostically (insensitive in early multiple sclerosis for subclinical brainstem lesions). Somatosensory-evoked, or visual-evoked, potentials may be useful in selected patients to ascertain a second lesion or show slowing, suggestive of central demyelination. CSF may show mild elevation in WBCs, predominantly lymphocytes. Measures of immune activity are sometimes but not always abnormal, including immunoglobulin G synthesis rate and the presence of oligoclonal bands (indicating antibody formation in the spinal fluid compartment).
    The prognosis for RRMS is better than most people expect. Various studies have shown a good prognosis over 10 years in a representative population without treatment. However, neurologic examination shows abnormalities after 10 years of disease in most patients. The diagnosis of benign multiple sclerosis is a retrospective one after 15 to 20 years of disease without measurable disability. This occurs in approximately 10% to 20% of patients with multiple sclerosis.
    Fox R, Bethoux F, Goldman MD, et al. Multiple sclerosis: Advances in understanding, diagnosing, and treating the underlying disease. Cleve Clin J Med. 2006; 73:91–102.
50
Q
  1. A 32-year-old Caucasian woman developed left retro-orbital pain, worse with eye movement, 1 week ago. Over the past 2 days she has lost central vision, has reduced perception of light brightness, and colors seem “washed out.” You examine her and find a normal fundus, a left relative afferent pupillary defect (RAPD), and visual acuity of 20/200 in the left eye. The following are correct, except:
    a. This condition usually improves over about 6 weeks
    b. Oral steroids are appropriate treatment
    c. This condition is sometimes associated with the emergence of multiple sclerosis
    d. Intravenous steroids with an oral steroid taper are the treatment of choice
    e. For visual acuity of 20/50 or better, steroid treatment has not been shown to be beneficial
  2. The presence of a relative afferent pupillary defect implies:
    a. An imminent early third nerve palsy
    b. Dampened efferent arm of the pupillary light reflex
    c. A particularly good prognosis
    d. A meningeal inflammatory component
    e. Involvement proximal to the Edinger-Westphal nucleus
  3. Testing that is valuable in such a patient includes:
    a. Syphilis serology
    b. Angiotensin-converting enzyme
    c. MRI of the brain and orbits with gadolinium
    d. Brainstem auditory-evoked potentials
    e. Retinal angiography
A
  1. b, 5. e, 6. c
    This patient has optic neuritis. The results of a large randomized double-blind treatment trial for acute optic neuritis compared intravenous methylprednisolone with prednisone taper, oral prednisone taper, and placebo. This study showed that recovery was hastened by intravenous methylprednisolone with oral prednisone taper but that visual outcome at 1 year was not significantly affected. Oral steroids were at best no better than placebo, and possibly worse.
    In about 80% of patients with acute optic neuritis, recovery occurs over weeks. This may be incomplete. Over a 15-year period after an episode of optic neuritis, multiple sclerosis occurs in about 50% of cases. Factors that increase the risk of multiple sclerosis developing include the presence of brain MRI lesions consistent with demyelination and abnormal CSF findings. In the Optic Neuropathy Treatment Trial, patients with visual acuity of 20/50 or better did not measurably benefit from intravenous steroids treatment.
    Relative afferent pupillary defect is a sign seen in optic neuritis. In this condition, there is involvement of the optic nerve with impaired response to light stimulation. The afferent arm of the pupillary light reflex is affected, in the side of the optic neuritis as compared to the normal eye. This leads to a relative reduction in signal traveling to the Edinger Westphal nucleus in the midbrain from the affected eye. Thus light shone into the affected eye causes a normal consensual light reflex. (Both the pupils constrict.) When the stimulus is shifted to the “good” eye, pupils remain constricted, and then when shifted back to the affected eye, there is relatively less “light,” so the pupils dilate (see also Chapter 1). This is not an indication of an effect on the third nerve, which is the efferent arm of the pupillary light reflex. This does not suggest a particular prognosis. This does not imply a meningeal component.
    MRI of the brain is ordered primarily to assess evidence and risk of progression to multiple sclerosis. On occasion, imaging of the optic nerve can confirm optic nerve involvement, but it is not critical for the diagnosis of optic neuritis. Rarely, retro-orbital or meningeal processes can cause sudden visual loss, simulating optic neuritis. On occasion, sinus cystic disease can cause impingement on or inflammation in the optic nerve. If there are one or more well-defined demyelinating lesions on MRI, the risk of multiple sclerosis goes from about 20% in 5 years to approximately 80% over 5 years. Multiple treatment trials in patients with a single episode of demyelination and MRI brain lesions have shown benefit from starting medicines that are FDA approved for multiple sclerosis treatment in such patients. In general, when the MRI does not show such lesions, treatment is not indicated beyond steroid therapy acutely.
    Unless the patient has a history suggestive of syphilis, syphilis serology is not useful. Angiotensin-converting enzyme testing is not useful in this situation unless there are other indications to suggest sarcoidosis. Brainstem auditory-evoked potentials are of low yield in this situation. There is no indication for retinal angiography in typical optic neuritis. It may be useful if there are atypical features such as a horizontal scotoma or very sudden painless onset suggesting a vascular etiology.
    Balcer LJ. Optic neuritis. N Engl J Med. 2006; 354:1273–1280.
    Beck RW, Cleary PA, Anderson MM Jr, et al. A randomized, controlled trial of corticosteroids in the treatment of acute optic neuritis. The Optic Neuritis Study Group. N Engl J Med. 1992; 326:581–588.
51
Q
  1. A 45-year-old woman with relapsing-remitting multiple sclerosis of 5 years’ duration develops new right leg weakness, with trouble walking more than 10 feet unsupported. She has no recent fever or infection and has not recently started a new medication. You examine her in the office and note increased reflexes in the right leg compared with prior examination, a Babinski sign on the right, and a right crossed adductor. Which of the following answers is correct?
    a. A Babinski sign is consistent with a spinothalamic tract demyelinating plaque
    b. Her weakness is due to a lower motor neuron disorder
    c. Intravenous methylprednisolone is the treatment of choice
    d. She should immediately begin working out harder
    e. A crossed adductor is a clear sign of cervical spinal cord disease
A
  1. c
    The patient is having an acute relapse. In this case there is a functional deficit interfering with walking, which is an indication for treatment rather than conservative management. Intravenous methylprednisolone usually given as a dose of 1000 mg daily for 3 to 5 days, with an oral steroid taper, is presently the treatment of choice for acute relapses. Other options include other forms of steroids, different dosing or duration of taper, not using a tapering dose, and high-dose steroids. The goal of treatment is to reduce the functional deficit and increase the speedy recovery of function.
    The Babinski sign is elicited by stroking the lateral border of the sole of the foot with a moderately sharp object. A normal response is that the great toe goes down (plantar flexion). A positive Babinski sign is when the great toe goes up, often with fanning of the other toes. A full “triple flexion” response is when the ankle dorsiflexes, the knee flexes, and the hip flexes in addition to the Babinski sign. The presence of a Babinski sign implies dysfunction in the corticospinal (pyramidal) tract between the cortex and the lumbar spinal cord. A positive Babinski sign is an indication of an upper motor neuron disorder (corticospinal tract) and not a lower motor neuron disorder (problem affecting the lower motor neuron from the anterior horn of the spinal cord to the neuromuscular junction).
    Increased exercise during an acute exacerbation has not been shown to be beneficial. However regular exercise in multiple sclerosis has been shown to improve quality-of-life measures and reduce fatigue and depression scales. A crossed adductor sign is seen when performing the patellar reflex. The opposite (crossed) leg adducts (moves medially from the hip). This implies a spread of reflex above the L4 level into L2 and L3 segments of the cord, indicating disinhibition of the reflex arc. This is another upper motor neuron sign.
    Frohman EM, Shah A, Eggenberger E, et al. Corticosteroids for multiple sclerosis: I. application for treating exacerbations. Neurotherapeutics. 2007; 4(4):618–626.
52
Q
  1. Your patient with newly diagnosed multiple sclerosis comes to you and wants to know a few things about this disease. The only statement that is correct in multiple sclerosis is that:
    a. Multiple sclerosis is more common further from the equator
    b. Drinking milk definitely prevents multiple sclerosis
    c. Her identical twin has a 100% chance of getting multiple sclerosis too
    d. Multiple sclerosis is definitely due to mononucleosis
    e. Multiple sclerosis is environmentally transmitted
A
  1. a
    Multiple sclerosis is more prevalent the further north or south of the equator one lives. The cause for this has not been established. Moving after the age of 12 does not seem to alter this risk. There is no evidence that drinking milk or other dietary measures affect the development of multiple sclerosis. However various epidemiologic studies suggest a link between vitamin D levels and multiple sclerosis, with lower vitamin D levels being correlated with higher risk of multiple sclerosis. Whether this is related to the cause of multiple sclerosis or common factors is unclear. Monozygotic (identical) female twins have a 33% chance of both being affected by multiple sclerosis if one develops multiple sclerosis. This indicates a mixture of genetic and nongenetic factors in the disease causation. Although mononucleosis has been linked to multiple sclerosis at times, the relationship is unclear and no definitive pathogenic relationship can be shown at present. The best evidence against environmental transmission of multiple sclerosis comes from adoption studies that do not show an increase in multiple sclerosis in children adopted into multiple sclerosis-affected families. Multiple sclerosis exacerbations are more likely to occur after infections, but are not related to any specific infection. Motor vehicle accidents and surgical procedures have not been linked to the onset or exacerbation of multiple sclerosis.
    Fox RJ, Bethoux F, Goldman MD, et al. Advances in understanding, diagnosing and treating the underlying disease. Cleve Clin J Med. 2006; 73:91–102.
53
Q
  1. The pathology of active multiple sclerosis can consist of all of the following, except:
    a. Involvement of cortical gray matter
    b. Involvement of white matter
    c. Loss of myelin from axons
    d. Axonal transections
    e. Profuse basophilic infiltration in the meninges
A
  1. e
    Multiple sclerosis is not associated with basophilic or eosinophilic infiltration of the meninges. Lymphocytes enter the CNS from the periphery. There are focal areas of demyelination (plaques) with glial cell infiltration and inflammatory cell accumulation. Macrophages are present in the core of the plaques. Areas of remyelination can be seen (“shadow plaques”). Oligodendroglial cells are reduced in the plaque core and increased at the periphery. Normal-appearing white matter may show loss of axons and some gliosis. Cortical gray matter is affected in multiple sclerosis, but this is poorly visualized with present imaging techniques and is thus under-recorded during life. White matter is typically affected in multiple sclerosis; myelin is removed from around axons, and early in lesion formation, axons are also injured and measurably transected.
    Lucchinetti CF, Parisi J, Bruck W. The pathology of multiple sclerosis. Neurol Clin. 2006; 23:77–105.
54
Q
  1. A 34-year-old man presents with ascending paralysis occurring 2 weeks after a diarrheal illness. On examination he has weakness of all four limbs, more distally than proximally, and deep tendon reflexes are absent. Analysis of CSF shows 1 μL WBCs (normal up to 5 lymphocytes/μL) and a protein level of 114 mg/dL (normal up to 45 mg/dL). Regarding this condition, which of the following is incorrect?
    a. Corticosteroids are not indicated
    b. Vital capacity should be measured frequently
    c. Hypercapnia on arterial blood gas is the most sensitive indicator of the need for intubation
    d. Blood pressure and heart rate monitoring is necessary in these patients
    e. Intravenous immunoglobulin therapy or plasmapheresis can be used in this case
A
  1. c
    Hypercapnia on arterial blood gas is not a sensitive indicator of the need for intubation in this setting.
    This patient has a progressive ascending paralysis after a diarrheal illness and albumino-cytologic dissociation evidence from CSF analysis, which is consistent with Guillain Barré syndrome (GBS), an acute inflammatory demyelinating polyneuropathy. Patients with this disorder should be hospitalized and may need intensive unit care, since they may develop respiratory failure, inability to protect the airway, and autonomic dysfunction, with labile blood pressure and cardiac arrhythmias. Therefore, these patients should have close cardiac and ventilatory monitoring with frequent evaluations of negative inspiratory force and vital capacity. Arterial blood gases are not accurate predictors of the need for intubation and mechanical ventilation, since hypoxia and hypercapnia occur late in the course of respiratory failure, once the patient is decompensating.
    The care of these patients include general supportive care, prevention of complications, rehabilitation, and specific therapies for the inflammatory process, which include plasmapheresis and intravenous immunoglobulin. Steroids play no role in the treatment of GBS.
    Hughes RA, Wijdicks EF, Barohn R, et al. Practice parameter: Immunotherapy for Guillain-Barre syndrome: Report of the Quality Standards Subcommittee of the American Academy of Neurology. Neurology. 2003; 61:736–740.
    Suarez JI. Critical Care Neurology and Neurosurgery, 1st ed. Totowa, NJ: Humana Press; 2004
55
Q
  1. Regarding embryonal nervous system development, which of the following statements is correct?
    a. The main embryonal layer giving rise to the nervous system is the endoderm
    b. The notochord is the main structure giving rise to the CNS
    c. The neural plate, through a process called neurulation, forms the neural tube
    d. The neural plate fuses to form the neural tube in all areas at once, simultaneously
    e. The notochord consists of ectodermal cells
  2. Regarding embryonal nervous system development, which of the following statements is correct?
    a. The rhombencephalon gives rise to the cerebral hemispheres
    b. The mesencephalon gives rise to the hypothalamus and thalamus
    c. The prosencephalon gives rise to the telencephalon, which ultimately forms the cerebral hemispheres
    d. The diencephalon gives rise to the midbrain
    e. The prosencephalon gives rise to the telencephalon, which ultimately forms the brain stem
  3. Regarding embryonal nervous system development, which of the following statements is correct?
    a. Vertebral bodies arise from ectodermal cells of the neural plate
    b. Neural crest cells give rise to the peripheral nervous system
    c. Failure of fusion of the posterior neuropore leads to disorders such as anencephaly
    d. Failure of fusion of the anterior neuropore leads to disorders such as spina bifida
    e. The notochord gives rise to chromaffin cells of the adrenal medulla
A
  1. c, 2. c, 3. b
    The main embryonal layer giving rise to the nervous system is the ectoderm. In early stages of nervous system development, a structure known as the neural plate forms. The notochord, a layer of mesodermal cells in contact with the ectoderm, induces formation of the neural plate from the ectoderm and later signals differentiation of various cell types mediated by inductive signals. The notochord later gives rise to the vertebral column.
    The neural plate forms a structure known as the neural tube through a process called neurulation. Neurulation involves proliferation and migration of ectodermal cells and invagination, folding, and fusion of the neural plate in a specific pattern. An important step in neurulation includes the formation of a midline groove along which the lateral margins of the neural plate fold. These lateral margins start to fuse in the center, so that for a period of time, there are openings at each end, the anterior and posterior neuropore. Fusion then reaches the neuropores, the anterior one first, then the posterior one, and the neural tube is thus formed. The ventral and dorsal aspects of the neural tube each give rise to specific cell populations and ultimately specific parts of the nervous system. As mentioned, mesodermal cells of the notochord provide signals for differentiation to cells in the ventral aspect of the neural tube. These signals include sonic hedgehog protein and bone morphogenetic proteins, of which several types have been identified.
    Neurulation occurs at 3 to 6 weeks’ gestation, and failure of crucial processes at any stage leads to a variety of abnormalities, collectively known as neural tube defects. As mentioned earlier, the neural plate undergoes fusion to form the neural tube in different areas at different times, and failure to fuse at each site results in specific defects. Abnormal rostral fusion at the anterior neuropore leads to abnormalities such as encephalocele or anencephaly, whereas abnormal caudal fusion (at the posterior neuropore) leads to disorders such as spina bifida.
    Following neurulation, the neural tube undergoes segmentation into three vesicles, in a process called specification, whereby different segments begin to acquire cell types and characteristics specific to the CNS structure that will eventually arise from them (malformations resulting from defects in this stage of development are discussed in subsequent questions). The three segments include the prosencephalon, mesencephalon, and rhombencephalon. The prosencephalon subsequently forms the telencephalon, which gives rise to the cerebral hemispheres, as well as the diencephalon, which forms the hypothalamus and thalamus. The rhombencephalon gives rise to the brain stem. Abnormalities during specification, which occurs at 5 to 6 weeks of gestation, lead to disorders such as septo-optic dysplasia (discussed in question 30). As the nervous system becomes more organized and specific areas more specialized, neuronal migration begins along specific routes, and abnormalities of neuronal proliferation and migration in turn lead to specific developmental abnormalities (discussed in subsequent questions).
    The peripheral nervous system (including the autonomic ganglia) forms from neural crest cells that are derived from the neural tube after it fuses. In addition to peripheral nervous system structures, neural crest cells give rise to the chromaffin tissue of the adrenal medulla and melanocytes.
    Swaiman KF, Ashwal S, Ferriero DM. Pediatric Neurology Principles and Practice, 4th ed. Philadelphia, PA: Mosby Elsevier; 200
56
Q
  1. Which of the following definitions of neural tube defects is incorrect?
    a. Meningocele—isolated meningeal protrusion
    b. Myelomeningocele—protrusion of spinal cord and meninges
    c. Diastematomyelia—splitting of the spinal cord into two portions by a midline septum
    d. Diplomyelia—duplication of the spinal cord
    e. Sacral agenesis—isolated absence of the sacral spinal cord
A
  1. e
    Sacral agenesis is absence of the sacrum, rather than absence of the sacral spinal cord, and is frequently associated with other malformations. The neural tube defects (NTDs) include meningocele, myelomeningocele, diastematomyelia, diplomyelia, and sacral agenesis. Caudal NTDs result from failure of fusion of the posterior neuropore at day 26 of gestation.
    Meningocele is isolated protrusion of the meninges into a bony defect within the vertebral column. This protrusion is typically covered by skin. It is not usually associated with neurologic deficits, and if neurologic deficit is found, myelomeningocele should be suspected.
    Myelomeningocele (also known as spinal dysraphism or rachischisis) is protrusion of potentially all layers of intraspinal contents through a bony defect: spinal cord, nerve roots, and meninges. Either the spinal cord may be exposed or a thin membrane may cover the protrusion. They most often occur in the lumbosacral region but can occur at any level. This is a clinically severe NTD associated with hydrocephalus, motor and sensory abnormalities of the legs, and bowel/bladder dysfunction. Myelomeningocele occurs in association with Chiari II malformations (discussed in question 21).
    Diastematomyelia is splitting of the spinal cord into two portions by a midline septum. Diplomyelia is duplication of the spinal cord and is distinguished from diastematomyelia by the presence of two central canals each surrounded by gray and white matter as in a normal spinal cord.
    Sacral agenesis, or absence of the whole (or in some cases parts of) the sacrum, classically occurs in association with a variety of other urogenital, gastrointestinal, and spinal cord abnormalities. It has been associated with maternal insulin-dependent diabetes. Autosomal dominant forms associated with homeobox gene mutations have been identified. Clinical manifestations range from mild motor deficits to severe sensory and motor deficits and bowel and bladder dysfunction.
    Management of NTDs involves surgical approaches and management of complications including hydrocephalus and bowel and bladder dysfunction. A search for associated malformations in other organs (such as the heart or kidneys) should always occur.
    Bradley WG, Daroff RB, Fenichel GM, et al. Neurology in Clinical Practice, 5th ed. Philadelphia, PA: Elsevier; 2008.
    Swaiman KF, Ashwal S, Ferriero DM. Pediatric Neurology Principles and Practice, 4th ed. Philadelphia, PA: Mosby Elsevier; 2006.
57
Q
  1. A 6-month-old baby is brought to the pediatrician for routine well-child visit. On examination, the infant is found to have a small, midline tuft of hair over the lower lumbar region. The child’s physical examination is otherwise entirely normal. On annual follow-up at 3 years of age, the child continues to have normal development and a normal neurologic examination. Which of the following statements regarding this child is correct?
    a. On the basis of the finding of this tuft of hair, it can be concluded that he will eventually have significant cognitive delay, even if he has developed normally up until the age of 3 years
    b. On the basis of the finding of this tuft of hair, it can be concluded that he will eventually have significant motor and cognitive delay even if he has developed normally up until the age of 3 years
    c. This tuft of hair signifies a possible underlying defect in the posterior bony component of the vertebral column
    d. This tuft of hair signifies the presence of a myelomeningocele
    e. On the basis of the finding of this tuft of hair, it can be concluded that abnormalities in the spinal cord will definitely be seen on imaging
A
  1. c
    This patient’s history and examination are consistent with spina bifida occulta. This is a defect in the bony components along the posterior aspect of the vertebral column. It can often be asymptomatic, but an abnormal conus medullaris and filum terminale are possible. The presence of a tuft of hair, implying underlying spina bifida occulta, does not necessarily imply impending cognitive or motor delay. In fact, when early neurologic development is normal, it will typically continue to be so. However, associated neurologic dysfunction may portend future neurologic impairment.
    When there is associated neurologic dysfunction in a child with a tuft of hair over the lumbar region but with no other evidence of neural tube defect, the disorder is named occult spinal dysraphism. In occult spinal dysraphism, a variety of developmental abnormalities may be seen involving the spinal cord or roots and posterior fossa, and associated findings may include dermoid or epidermoid cysts, intraspinal or cutaneous lipomas, and tethered cord. Diastematomyelia, or splitting of the spinal cord, may also be seen. Rarely, a sinus tract connects the dura with the surface of the skin. In occult spinal dysraphism, neurologic manifestations vary widely and may range from minimal motor deficits and ankle hyporeflexia to bowel and bladder dysfunction, sensory loss, and paraparesis or paraplegia. Although patients may be initially asymptomatic, these neurological deficits can develop suddenly and be irreversible.
    Swaiman KF, Ashwal S, Ferriero DM. Pediatric Neurology Principles and Practice, 4th ed. Philadelphia, PA: Mosby Elsevier; 2006.
    Volpe JJ. Neurology of the Newborn, 4th ed. Philadelphia, PA: WB Saunders; 200
58
Q
  1. Which of the following is not a risk factor for the occurrence of neural tube defects?
    a. Male gender
    b. Folate deficiency
    c. Exposure to retinoic acid
    d. Exposure to valproic acid
    e. Maternal diabetes
A
  1. a
    Several risk factors for neural tube defects (NTDs) have been identified. NTDs are more common in females. Folate is involved in various pathways of nucleic acid synthesis and DNA methylation reactions, and maternal folate deficiency is a well-established risk factor for NTD. Therefore, prenatal and perinatal maternal supplementation with 0.4 mg folic acid is recommended. Teratogens associated with NTDs include retinoic acid (vitamin A or the acidic form, tretinoin, found in acne medications). Other teratogens associated with NTDs include antiepileptics, particularly valproic acid and carbamazepine, which may lead to NTDs by affecting folate metabolism. Other risk factors for NTDs include maternal diabetes and history of a prior pregnancy resulting in an infant with an NTD.
    Swaiman KF, Ashwal S, Ferriero DM. Pediatric Neurology Principles and Practice, 4th ed. Philadelphia, PA: Mosby Elsevier; 2006.
59
Q
  1. In the setting of a fetus with a neural tube defect (NTD), which of the following statements regarding prenatal diagnosis is incorrect?
    a. α-Fetoprotein level is elevated in maternal serum
    b. α-Fetoprotein level is elevated in amniotic fluid
    c. Acetylcholinesterase is elevated in amniotic fluid and increases sensitivity and specificity of NTD screening
    d. There is little utility in prenatal ultrasonography in the detection of NTDs
    e. When a NTD is detected prenatally, karyotyping to assess for trisomies and other genetic defects may be indicated
A
  1. d
    Prenatal ultrasonography is used in detecting neural tube defects (NTDs) and characterizing them. Prenatally, in the setting of most NTDs, serum maternal α-fetoprotein level is elevated. α-Fetoprotein is a normal component of fetal CSF, and leakage into the amniotic fluid from an open neural tube leads to elevated amniotic α-fetoprotein level. The extent of elevation correlates with the severity of the NTD. Elevations in amniotic fluid acetylcholinesterase levels also occur, and combined with an elevated α-fetoprotein level, increase sensitivity and specificity in prenatal screening. Ultrasonography also can be used to detect NTDs and characterize them. Prenatal MRI is also used in some cases to assess the extent of the abnormality, aiding in prognostication of neurologic function in life. When an NTD is detected with screening, and/or other abnormalities are detected on ultrasonography, fetal karyotyping to assess for trisomy 13, 18, and others may be done to assist in management of the pregnancy.
    Swaiman KF, Ashwal S, Ferriero DM. Pediatric Neurology Principles and Practice, 4th ed. Philadelphia, PA: Mosby Elsevier; 2006.
60
Q
  1. While you are working your 4th of July shift in the emergency department, a 62-year-old farmer with mental status changes and generalized weakness is brought by his wife. The patient has been having low-grade fevers, malaise and back pain, body aches, and headaches for the past 10 days; however, he did not want to come to the hospital. Over the past 5 days, he has developed bilateral hand tremors and difficulty walking. About 3 days ago, he became confused, and today, he was noticed to be unable to move his legs. On examination, he is lethargic and confused and noticed to be flaccid in his lower extremities, with areflexia. Which of the following is the most likely diagnosis?
    a. Tuberculous meningitis and Pott’s disease
    b. Subacute combined degeneration of the spinal cord
    c. Neurosyphilis with tabes dorsalis
    d. West Nile encephalitis
    e. HSV encephalitis
A
  1. d
    This patient has West Nile virus (WNV) encephalitis, which is an arboviral infection that spreads in the summer months, can occur in epidemics, and is transmitted by mosquitoes of the genus Culex. Most infected patients are asymptomatic, up to 20% may show signs of a febrile viral illness, and less than 1% develop a severe neurologic presentation; elderly patients are particularly at risk for neurologic disease and death. Patients with this neurologic presentation will have manifestations of encephalitis; however, WNV can also invade the anterior horn cells leading to flaccid weakness with areflexia, similar to poliomyelitis. WNV infection may also manifest with cranial neuropathies and tremors.
    WNV encephalitis is diagnosed by serology and detection of immunoglobulin M (IgM) antibodies in the CSF. CSF PCR is less sensitive, but is diagnostic when positive. MRI is nondiagnostic. Treatment is supportive.
    This patient does not have findings to support the other conditions listed.
    Bradley WG, Daroff RB, Fenichel GM, et al. Neurology in Clinical Practice, 5th ed. Philadelphia, PA: Elsevier; 2008.
61
Q
  1. Mycobacterium tuberculosis may invade the spinal column. Which of the following is incorrect?
    a. The mid-thoracic region is most commonly affected
    b. Back pain and muscle spasms are common symptoms
    c. Spinal disease usually starts in the spinous processes
    d. Kyphotic deformities are common
    e. Long-standing disease may produce vertebral body destruction and collapse
A
  1. c
    Tuberculosis can affect the spine, more specifically the vertebral column (Pott’s disease). The most commonly affected region is the midthoracic spine. Patients frequently present with constitutional symptoms, back pain, and muscle spasms, sometimes with findings of compressive radiculopathy or myelopathy.
    Mycobacterium tuberculosis invades the body of the vertebral column, usually starting in the anterior region of the vertebral bodies, not at the spinous processes. The infectious process tends to disrupt the bone and the intervertebral spaces, leading to vertebral body destruction and collapse, as well as vertebral column deformities with kyphosis and scoliosis.
    Bradley WG, Daroff RB, Fenichel GM, et al. Neurology in Clinical Practice, 5th ed. Philadelphia, PA: Elsevier; 2008.
62
Q
  1. Typical symptoms of multiple sclerosis may include all of the following, except:
    a. Tingling, burning, aching, or “numb” sensations
    b. Weakness of the legs
    c. Double vision
    d. Distal limb atrophy
    e. Ataxia
A
  1. d
    Multiple sclerosis is typically an upper motor neuron or CNS disorder and does not cause muscle atrophy. However there are reported cases of some peripheral involvement and muscle atrophy in late multiple sclerosis, but these would be atypical features and suggest other diagnoses. Paresthesias of various types including the above as well as focal areas of pruritus or a sensation of wetness are common in multiple sclerosis. These can occur anywhere on the body. Weakness and gait difficulties are key common symptoms in multiple sclerosis, but do not occur in every patient. Double vision can occur and is usually due to brainstem involvement. Gait or limb ataxia occurs commonly and can be due to cerebellar, cerebellar connection, or sensory tract involvement in multiple sclerosis.
    Noseworthy JH, Lucchinetti C, Rodriguez M, et al. Multiple sclerosis. N Engl J Med. 2000; 343:938–952.
63
Q
  1. Your 34-year-old African-American female patient with relapsing neurologic symptoms has spinal fluid testing. Which of the following findings would prompt you to consider diagnoses other than multiple sclerosis?
    a. Elevated kappa chains
    b. WBC count of 22 (normal 0 to 5 cells/mm3)
    c. Neutrophilic pleocytosis
    d. Elevated immunoglobulin G synthesis rate
    e. Normal CSF immunologic parameters
A
  1. c
    White cells can be elevated in the CSF in multiple sclerosis. Usually there is a predominance of lymphocytes and no more than 50 cells/mm3. A neutrophilic pleocytosis would be unusual and might suggest another diagnosis. For example this can be seen in Devic’s disease at times (neuromyelitis optica; see discussion to questions 14 to 18). Elevated kappa chains are an indication of antibody formation in the CNS compartment and can be seen in multiple sclerosis. Immunoglobulin G synthesis rate elevation is one measure of increased intrathecal elaboration of antibody and can be seen in multiple sclerosis. Some patients with multiple sclerosis can have normal CSF. The frequency of this finding varies, but a normal CSF does not “rule out” multiple sclerosis.
    Noseworthy JH, Lucchinetti C, Rodriguez M, et al. Multiple sclerosis. N Engl J Med. 2000; 343:938–952.
64
Q
A